Sie sind auf Seite 1von 45

SMALL INTERNSHIP

CONTENTS
NEUROLOGY - NEUROSURGERY ............................................... .................................................. ..........
7
PSYCHIATRY ......................................................... .................................................. .............................
121
EAR - NOSE AND THROAT DISEASES ............................................ ................................... 153
EYE DISEASES ................................................ .................................................. ................ 187
UROLOGY ..................................................... .................................................. ................................ 213

DERMATOLOGY ................................................. .................................................. ....................... 245


ORTHOPEDICS ................................................. .................................................. .............................
291
CHILD SURGERY ................................................... .................................................. ................. 329
CHEST AND CARDIOVASCULAR SURGERY, RADIOLOGY .......................................... ........................... 357

PHYSICAL THERAPY AND REHABILITATION .............................................. ...............................................


383
ANESTHESIOLOGY ......................................................... .................................................. ...................
405
PUBLIC HEALTH, MEDICAL STATISTICS, DEONTOLOGY ...........................................
................................. 417
NEUROLOGY - NEUROSURGERY
NEUROLOGY - NEUROSURGERY
one. A 35-year-old male patient was admitted with a contraction in the cold and difficulty in opening his
hands. On physical examination, frontal baldness, bilateral cataract, facial weakness and thenar
percussion myotony.
What is the preliminary diagnosis for this patient? (September 2011)
A) Myasthenia gravis
B) Myotonia congenita
C) Myotonic dystrophy
D) Oculopharyngeal muscular dystrophy
E) Neuromyotonia
Myotonic dystrophy Autosomal dominant inheritance. The contracted muscle cannot relax (action myotony).
40–50s
It has a typical facial appearance. Frontal baldness, ptosis, low jaw, depressed cheek,
neck and shoulder girdle muscles have atrophy. Cataract is present in 80% of cases. Testicular atrophy,
diabetes. There is weakness and atrophy in the distal extremities.
(Answer C)
2nd. A 65-year-old diabetic and hypertensive patient was admitted to hospital with acute developing
quadriplegia.
Fetching. In her neurological examination, aphonia was limited by horizontal eye movements, but vertical
eye movements were intact, and blinking response to verbal stimuli
tracked. Which of the following syndromes should be considered first in this patient? (September 2011)
A) Locked-in syndrome
B) Dorsal mesencephalic syndrome
C) Guillain-Barre syndrome
D) Dorsal pontin syndrome
E) Millard-Gubler syndrome
TUSTIME ALL TASS QUESTIONS
7
SMALL INTERNSHIP
In Locked-in syndrome, the destruction of the ventral brainstem where RAS is preserved causes the patient
to be silent and quadriplegic, but not non-comatous. All four extremities and lower
Complete paralysis of the cranial nerves is observed, but is not related to impaired consciousness. Patients
with Lockedin syndrome have quadriplegia and anarthria, but consciousness and reason are variable.

file:///C|/Users/User/Downloads/small%20intern.htm[06-Aug-19 12:48:59 AM]


percentages. The patient is alert, but does not speak or move, and responds slightly to stimuli. The lesion
usually involves the middle pons and is parallel to the lateral movements of the face.
result. Such a lesion is responsible for the somatosensory pathways responsible for wakefulness and the
removal of the eyelids by disrupting the resulting neuronal system and even some midbrain structures.
and give the impression that it is awake. Lesion corticobulber and corticospinal tracts
and causes the patient to be unable to speak, with no response other than vertical gaze and blink.
Communication through eye movements and blinkers, albeit difficult
It can be established. The primary cause of Locked-in syndrome is brainstem stroke (86%), but can also be
observed after trauma (14%).
(Answer A)
3. A 6-year-old boy was admitted to the hospital with a 20-day history of headache.
poop. On physical examination, bilateral papillary stasis and computed tomography showed a mass filling
the fourth ventricle causing hydrocephalus.
What is the preliminary diagnosis for this patient?
A) Medulloblastoma
B) Craniopharyngioma
C) Acoustic neurinoma
D) Meningioma
E) Pinealom
medulloblastoma
It is the most common malignant intracranial tumor in the first decade. Often the cerebellar comes out of
the vermis.
Propagation to CSF is common. hydrocephalus
(Answer A)
4. A 55-year-old male patient who underwent surgery for a localized tumor in the left occipital lobe
patolojik incelemede yassı hücreli karsinom metastazı saptanmıştır. The organ that should be examined first
in terms of primary focus in this patient
Which of the following? (May 2011)
A) Kidneys
B) Lung
C) Stomach
D) Esophageal
E) Column
8
TUSTIME ALL TASS QUESTIONS
NEUROLOGY - NEUROSURGERY
FAQ METASTATIC TUMORS At the beginning of primary organ cancers metastasizing to central nervous
system
Lung Malignancies come. Breast cancers and malignant melanoma metastases follow this order. In the
fourth order, cases where the primary focus of brain metastasis cannot be detected are listed.
Breast CA in women and Lung CA (SCLC) in men is the most common metastatic tumor.
(Answer B)
5. Headache and sleep tendency in the last 12-24 hours
Fever was 38.2 ° C, nuchal rigidity was determined and lethargic and confused. Focal or other lateralizing
There was no neurological finding. Cerebrospinal fluid examination revealed 100 lymphocytes per mm3,
with normal glucose and protein values.
Which of the following is the most likely diagnosis for this patient? (May 2011)
A) Bacterial meningitis
B) Tuberculous meningitis
C) Viral meningitis
D) Fungal meningitis
E) Carcinomatous meningitis
Clinically, meningitis was brought to the emergency room and the patient had more than 200 cells in his
CSF.
Lack of lymphocyte character and normal sugar and protein values are in favor of viral meningitis.
CSF Pressure
Cell / mm of water

file:///C|/Users/User/Downloads/small%20intern.htm[06-Aug-19 12:48:59 AM]


mm3
9 Cell type Protein mg / dll Sugar
Normal 150 <5 Lymphocyte 15-45 2/3 blood sugar Bacterial Increased> 500 PNL> 100 <2/3 blood sugar
Tbc * and fungal Increased 200-500 Lymphocyte> 100 <2/3 blood sugar Viral Increased <200 Lymphocyte
Normal or
light high
Normal
(Answer C)
6. A twenty-three-year-old female patient with a drop in her eyelids and double vision while studying at
night in the evening for a few weeks, passed away after taking a break and resting
but after a while, he complains again about the same situation. The ophthalmologist could not detect a
refractive error in his eyes. Bilateral mild neurological examination
There is no pathology except ptosis. Which of the following should be considered first in this patient? (May
2011)
A) Masked depression
B) Guillain-Barre syndrome
C) Myasthenia gravis
D) Botulinum poisoning
E) Multiple sclerosis
TUSTIME ALL TASS QUESTIONS
MYASTENIA GRAVIS Neuromuscular junction disease.
There are antibodies developed against autoimmune post-synaptic acetyl choline receptors. Muscle
weakness increased by repetitive movements and improvement of this weakness by anticholinesterases are
typical.
It can begin at any age. It is common in women aged 20-40 years. 8-10% with thymoma. Ach receptors in
the neuromuscular junction in myasthenia gravis
antibody development and decrease in the number of receptors. The disease often begins with ocular
symptoms and most often with unilateral ptosis. Later
double vision is added. Patients with these findings alone are called ocular MG. (10%).
Difficulty in speaking, difficulty in swallowing, difficulty in chewing, difficulty in breathing are bulbar findings.
Although early fatigue may begin earlier in the extremities, bulbar findings are often
occurs before.
Diagnosis:
Anticholinesterase drugs are used. Edrofonium chloride (Tensilon) is injected. Visible
If there is improvement (improvement in ptosis) the test is positive. EMG is used in diagnosis.
Treatment: Acetylcholinesterase inhibitors (pyridostigmine), steroids, plasmapheresis, immunosuppressants,
azotio-
prin, IgG, thymectomy.
(Answer C)
7. Which of the following cerebrospinal fluid examination findings is diagnosed with multiple sclerosis?
is important for? (December 2010)
A) Protein increase
B) Oligoclonal band positivity
C) IgG index below 0.6
D) Mononuclear cell increase
E) Presence of myelin degradation products
SMALL INTERNSHIP
TUSTIME ALL TUS QUESTIONS 10
MULTIPL SCLEROSIS
Definition
MS is a demyelinating disease in young adults.
symptoms
Weakness, paresthesia, retrobulbar neuritis, diplopia, urinary incontinence due to sphincter problems
The disease goes with relapses and remissions.
Myelin basic pretein (MBP) is the most important antigen for T cells.
MBP – reactive T lymphocytes are found in the blood and CSF of the patients.
In the sera of MS patients, it is also possible against myelin components (myelin oligodendrocytes

file:///C|/Users/User/Downloads/small%20intern.htm[06-Aug-19 12:48:59 AM]


glycoprotein
MOG) antibodies.
Optic nerve lesions are among the most common initial findings in MS.
NEUROLOGY - NEUROSURGERY
Figure: Bilateral Internuclear ophthalmoplegia (MLF lesion)
TUSTIME ALL TASS QUESTIONS
11th
SMALL INTERNSHIP
I. Motor neurons, sensory neurons, autonomic CNS neurons.
2nd. The motor neuron does not involve the peripheral nerves.
Diagnosis
Presence of plaques on MRI
Evoked potentials
The demonstration of oligoclonal bands in CSF are laboratory findings supporting the diagnosis of multiple
sclerosis.
All questions of TUSTIME (Answer B). A 42-year-old alcoholic patient with malnutrition
with the emergency department. Neurological examination revealed nystagmus and conjugate
lysis, trunk ataxia, global confusion and memory impairment.
Which of the following is the most likely diagnosis for this patient? (December 2010)
A) Tobacco-alcohol amblyopia
B) Subacute combined degeneration
C) Alcoholic cerebellar degeneration
D) Wernicke – Korsakoff syndrome
E) Strachan syndrome
WERNICKE - KORSAKOFF SYNDROME (WKS)
Wernicke's Encephalopathy and Korsakoff Psychosis, which were described separately in the last decades of
the 19th century,
They have been described as Wernicke-Korsakoff Syndrome (WKS) because of their common etiologic,
clinical and histopathological features.
Wernicke's encephalopathy (polioencephalitis hemorrhagica superioris) is characterized by acute or subacute
ocular findings, ataxia and confusion. In Korsakoff Psychosis, other
while cognitive functions are relatively preserved, significant impairment is in the memory area. Korsakoff
Psychosis is frequently seen with Wernicke's Encephalopathy. Korsakoff Syndrome, Wernicke's
Encephalopathy
It would not be a wrong definition to characterize as psychic symptoms added in the course of the course.
NEUROLOGY - NEUROSURGERY
Avitaminosis and especially thiamine (B1) deficiency are responsible for the etiology. Autopsy studies
showed a selective reduction in the activity of thiamine-dependent enzymes. Also poor diet with B1
WKS findings were observed in animals fed. Syndrome is thought to occur due to inadequate diamine
intake. This syndrome is most commonly observed in chronic alcoholics,
the direct neurotoxic effect of alcohol, together with the limitation of thiamine intake, is responsible for the
pathogenesis.
suspicion.
Clinical signs and symptoms
The most common symptoms of the syndrome are impaired mental status, oculomotor findings and
unstable
It is walking.
Mental Findings:
Confusion is the most common mental symptom in acute WKS syndrome. The patient is apathic.
tation has disappeared. Ataxia and oculomotor findings accompany the confusional state. This treatment
If the disease is not started in this period, the disease progresses and persistent symptoms occur.
Confusional status
it becomes a serious amnesia.
Anterograde amnesia:
In this period, no pathological findings were found related to other cognitive areas of the patient,
severe disorder is close to itself as a result of inability to learn new verbal and non-verbal information.
the inability to remember the names and faces of newly-met people,

file:///C|/Users/User/Downloads/small%20intern.htm[06-Aug-19 12:48:59 AM]


it shows that the current events are never forgotten. Inability to learn new information
It is the result of encoding defect in the first step of the recording processes.
Apart from memory problems, sometimes frontal finding is found in the form of problem solving and concept
formation difficulties.
gulle Another cause of memory impairment is recorded in memory functions
the frontal system involved in retrieving information. Intellectual abilities except amnesia
It is normal.
Retrograde amnesia:
WKS cases are social events in a period extending back from the onset of the disease,
they cannot remember information about themselves. How far back from the onset of the disease
information of the period is better remembered. So the sick childhood and early adulthood
He cannot remember his recent past from the onset of the disease.
The severity of retrograde amnesia is correct in the failure of tests showing frontal lobe function.
It was found proportionally. Frontal in planning and initiating systematic memory research
The lobe is important. Information learned as premorbid due to this disorder is restored
and retrograde amnesia occurs.
Eye findings:
Nystagmus is the most common finding. Horizontal nystagmus is often seen in the direction of view. Some
vertical or rotational nystagmus may also be observed. Limitation of external eye movements first
as the eye shows itself in the horizontal plane. Later on the vertical plan
limitation of eye movements is added to the table. Restriction of eye movements, asymmetric bilateral
lateral
from conjugate gaze restriction, which may also be present in rectal paralysis and vertical gaze to total
ophthalmoplegia
up to a fan. Reduced pupil light response may be seen but this
total reflex failure is not a detectable finding. Moderate ptosis accompanying clinical picture
Can.
TUSTIME ALL TASS QUESTIONS
13
SMALL INTERNSHIP
Ataxia:
There is trunk ataxia in the foreground. The patient walks unevenly, wobbling, opening his steps.
This may be accompanied by extremity ataxia. In addition to the involvement of the cerebellum, vestibular
system
is responsible for the findings.
Diagnosis
Good anamnesis, strict diet, alcohol consumption and consumption
required.
(Answer D)
9. In a patient with head trauma who was brought into a coma, painful stimuli were opened.
no response to honey stimulus and a normal flexor response to painful stimulus
It was. What is the Glasgow coma scale of this patient?
(E: eye opening, M: motor response, V: verbal) (December 2010)
A) E1 M3 V1
B) E2 M4 V1
C) E2 M5 V2
D) E2 M3 V3
E) E4 M5 V6
The degree of coma can be determined according to the Glasgow Coma Scale (GCS). Accordingly, GCS: 13–
15 is considered to be minimum, GCS: 9–12 is moderate, GCS≤8 is considered as severe brain injury and
GCS <5 deep coma.
The diagnosis of brain death can be made clinically by various supporting tests and radionuclide imaging
methods.
Glasgow Coma Scale
Eye opening score
Spontaneous -> 4 Orders -> 3
With painful stimulus -> 2 None -> 1

file:///C|/Users/User/Downloads/small%20intern.htm[06-Aug-19 12:48:59 AM]


Motor response Normal limb movements with orders -> 6
Localization of painful stimulus -> 5 Reaction to painful stimulus -> 4
Flexion to painful stimulus -> 3 Extension to painful stimulus -> 2
No answer to pain -> 1 Oral answer
Oryante – name, age -> 5 Confusion -> 4
Improper speech -> 3 Pointless grunt -> 2
No replies -> 1
(Answer Z)
14
TUSTIME ALL TASS QUESTIONS
NEUROLOGY - NEUROSURGERY
10. In which direction did the paralytic eye shift in a patient with oculomotor paralysis?
(December 2010)
A) Outward – downward B) Inward
C) Up D) Inward – down E) Inward – up
Oculomotor nerve (motor, parasympathetic)
Levator palpebra superior, medial, superior and inferior rectus, inferior oblique muscles and pupils
parasympathetic innervation.
Ptosis when not working. mydriasis. loss of light reaction, upward and downward gaze of the eye
restriction is seen, the eye shifts outward. Isolated III. most common causes of nerve paresis; aneurysm
or vascular diseases such as diabetes, hypertension, atherosclerosis.
Pupillary is mydriatic in compressive lesions such as aneurysm and tumor. (parasympathetic fibers
oculomotor
In the case of compression, parasympathetic fibers are affected firstly. Dialogue
In bet and other vascular lesions the pupil is usually unaffected.
(Answer A)
11th. A 40-year-old male patient suffered a serious injury during a traffic accident.
but there is a complaint of localized neck pain in the right.
Pain gradually increased and spread to the ipsilateral head within the following 2 days.
learning. Meanwhile, the left side strength develops. Known risk factors
On examination, the patient had mild ptosis and miosis on the right and hemipa on the left.
rezi and hemihipoes- thesis.
Which of the following is the most likely diagnosis for this patient?(April 2010)
A) Posttraumatic subarachnoid hemorrhage
B) Posttraumatic intracerebral hemorrhage
D) Carotid artery dissection
C) Cardioembolic stroke
E) Cervical medulla spinalis injury
Traumatic internal carotid artery (ICA) dissection, penetrating to the area where the carotid artery passes,
blunt
or iatrogenic trauma. The most common cause of dissection is blunt neck
And trauma. Immediately after carotid artery infusion, trauma to the region adjacent to the C1-C2 vertebra
is very sensitive. In the upper cervical region, lateral rotation over the transverse process of the cervical
vertebra
In addition, hyperextension trauma is a common mechanism in IKA dissection. dissec-
cations usually pass from cervical to petrosis of carotid artery, from cavernous to petrosis
occurs in parts
TUSTIME ALL TASS QUESTIONS
15
SMALL INTERNSHIP
In the case of carotid artery trauma, 50 percent of the cases come without any external findings. be-
Lymphs occur after a waiting period of 2-3 weeks. The most common symptom is severe head and neck.
pain is. The most important finding in posttraumatic dissection is ischemic symptoms.
Stroke is usually the result of embolism from the lumen of the dissected artery to the intracranial vessels,
and is often a result of moderate
towards the rebral artery. Unilateral motor and / or sensory loss, aphasia, amorosis fugas, incomplete
Horner syndrome (oculosympathetic paralysis), ptosis without anhidrosis and myosis (around ICA)

file:///C|/Users/User/Downloads/small%20intern.htm[06-Aug-19 12:48:59 AM]


external carotid artery plexus that innervates the facial sweat glands due to involvement of the plexus)
may develop transient ischemic attack, pulsatile tinnitus
Poor amplitude, but repetitive and long-range effects, such as a hand massager applied to the neck
IKA may cause dissection and stroke. Especially in motorcycle accidents
protection helmets, belt wrap lengthwise can lead to compression and dissection of the carotid artery
Carotid artery dissection in patients without an explanatory cause in brain tomography
It should be maintained. Magnetic resonance (MR) and magnetic resonance angiography (MRA) for
diagnosis and follow-up
It should be done. Especially MRA is the most popular noninvasive method in the diagnosis of carotid artery
dissection.
(Answer D)
12. A 28-year-old female patient with acute vision loss in her right eye
emergency department. Relative afferent pupil defect on right, visual
and severe central scotoma.
Where is the lesion in this patient? (September 2009)
A) Optic nerve
B) Optical chiasma
D) Lateral geniculate body
C) Optical tract
E) Occipital cortex
Optic neuritis is the first conceivable condition in a young female patient with acute vision loss. Multiple
sclerosis
30% of patients with optic neuritis.
(Answer A)
13. Unilateral retrobulbar neuritis is the most common of the following? (September 2009)
A) Diabetes mellitus
B) Vitamin deficiency
D) Late neurosyphilis
C) Leber's optical atrophy
E) Multiple sclerosis
The definition of retrobulbar neuritis is quite typical for multiple sclerosis. Leber optic atrophy is the most
common
is the cause of hereditary optic atrophy. Mitochondrial inheritance. Mitochondrial inheritance
is the first associated disease.
(Answer E)
16
TUSTIME ALL TASS QUESTIONS
14. Spontaneous subarachnoid hemorrhage occurs around the age of fifty.
which is the reason? (September 2009)
A) Arteriovenous malformation
B) Brain tumor
C) Aneurysm rupture
D) Atherosclerosis
E) Coagulopathies
Subarachnoid Hemorrhage
It is diagnosed with sudden, very severe headache, coma, stiff neck and bloody CSF. Bleeding is usually
caused by Berry aneurysms located in the head base around the Willis polygon.
Caused. The most common aneurysm is located between the ant.cominikan artery and the ant.cerebral
artery.
Vascular malformations (arteriovenous malformations, cavernoma) are the most common cause in children.
Increased intracranial pressure causes increased consciousness and headache, nausea, vomiting and
papilledema. Later, arterial vasospasm, edema and aneurysmal hemorrhage
focal neurological losses may occur when it passes into neighboring brain tissue. Diagnosis:
• IT
• Lumbar puncture
• Angiography
Treatment

file:///C|/Users/User/Downloads/small%20intern.htm[06-Aug-19 12:48:59 AM]


• Surgical or endovascular closure of aneurysm
• Nimodipine for arterial vasopasm after SAH
NEUROLOGY - NEUROSURGERY
TUSTIME ALL TASS QUESTIONS
17
SMALL INTERNSHIP
ALL TUSKY QUESTIONS (Answer C). A 54-year-old patient who was admitted to the emergency department
with acute ischemic stroke due to mitral valve disease had normal cranial computed tomography. It has
been learned that 1 hour has passed since the onset of the attack. Physical examination revealed right
hemiplegia and aphasia. What is the immediate treatment for this patient? (April 2009)
A) Thrombolytic therapy
B) Surgical cardiac thrombectomy
C) Surgical thrombectomy for a blocked cerebral vessel
D) Clinical follow-up with antiaggregant / anticoagulant therapy
E) Rehabilitation
Thrombolytic (t-PA) should be given in the first 3 hours in ischemic stroke.
(Answer A)
16. In a few minutes, which describes the possible loss of strength in the left arm and leg with the
underlying etiology, which is given together? (April 2009)
A) Paraplegia Encephalitis
B) Hemiplegia Cerebral tumor
C) Paraplegia Cerebrovascular event
D) Tetraplegia Cerebral tumor
E) Hemiplegia Cerebrovascular event
The rapid development of the event brings to mind the serobrovascular event. Since left hemiplegia was
described in the patient,
Ischemic event affecting the right arterial cerebri media is the first diagnosis.
(Answer E)
NEUROLOGY - NEUROSURGERY
17. The 36-year-old patient, who was being followed up for carbon monoxide intoxication, was able to
follow simple orders, but instead of answering the questions, repeating the question exactly. Which of the
following is the most likely diagnosis for this patient? (April 2009)
A) Broca's aphasia
B) Transcortical motor aphasia
C) Wernicke aphasia
D) Transcortical sensory aphasia
E) Transmission aphasia
Transcortical sensory aphasia: It is caused by damage at the posterior of the Wernicke area where the
parietal and temporal regions meet. Apart from its good repetition ability, its characteristics are very similar
to Wernicke aphasia. Transcortical motor aphasia: This type of aphasia is very similar to Broca's aphasia
with the exception of preserved repetition and reading ability. Another feature that is different from Broca's
is that the patient speech resembles stuttering. The lesion is the superior and anterior region of the Broca
area and is at the boundary of the perisylvian area. Dysarthria due to weakness of the oral organs is
observed in patients with broca and especially transcortical motor aphasia.
Broca's aphasia: The lesion is left lateral frontal, prerolandic, supra-
anterior third convolution which is immediately anterior to the precentral gyrus.
The lesion is at the border of the upper part of the middle cerebral vessels, in some cases
it extends posteriorly to the rietal area. The characteristics of the patient speech are arrested,
grammeless, telegraph type without prosody, melody and gravitational attachments. Understands what is
being told and
be aware of the wrong.
Conduction aphasia: Lesion The pathway of white matter connecting the Broca and Wernicke area
(arcuate facliculus) is the most prominent feature of the weakness of repetition. Although
Although it is known as fluent aphasia type, the more anterior the lesion, the more arrest the speech.
Wernicke Aphasia: lesion damage within the Wernicke area, or nearby temporal and parietal region
can. It is the central posterior third of the superior temporal gyrus. Lesion Wernicke
and inferior parietal area, fluent speech and phonemic paraphasia are observed.

file:///C|/Users/User/Downloads/small%20intern.htm[06-Aug-19 12:48:59 AM]


(Answer B)
18. In a traffic accident, the car hit the power pole from the front. Driver, seat belt
taken to hospital in unconscious state.
There was no lesion and no cranial fractures were detected on the radiographs.
The most likely finding in the head as a cause of death in a forensic autopsy
Which of the following? (April 2009)
A) Subdural bleeding
B) Epidural bleeding
C) Subarachnoid hemorrhage
D) Intracerebral hemorrhage
E) Cerebral laceration
TUSTIME ALL TASS QUESTIONS
19
SMALL INTERNSHIP
20 TUSTIME ALL TASS QUESTIONS Subdural Hemorrhage
• It is seen in 5% of patients with head trauma.
• Children and the elderly are affected more frequently.
• Occurs as a result of rupture of the bridge veins (between the cortex and venous sinuses).
• Sudden acceleration / deacceleration formation mechanism. There is a compression-related neurological
loss after bleeding.
• 20% of patients have a condition called chronic subdural effusion. Especially in elderly patients, there is
bleeding to the subdural distance after trauma. Plasma with blood osmotic effect
It pulls. Subdural effusion develops in the altar. Neurological loss and dementia due to pressure
It is seen.
• The earliest evidence of bleeding is CT.
Venous hemorrhage to the subdural distance appears to be a hyperdense structure parallel to the brain
convexity. Treatment: Subdural bleeding is drained.
Subdural bleeding
Epidural Hemorrhage Often accompanied by fractures of the head bones.
A.meningia is due to media bleeding. CT appears as lens-shaped (biconvex).
In 20% of cases, it forms a lucid interval. 2-3 hours after head trauma
then it can form the outgoing table with the sudden closure of consciousness. Diagnosis:
• CT Treatment:
• Surgery is evacuated
Epidural and subdural bleeding
(Answer A)
NEUROLOGY - NEUROSURGERY
19. In a patient with acute hyperextension trauma to the cervical vertebrae, the loss of strength in the
upper extremities was higher than in the lower extremities.
Which of the following should be considered? (September 2008)
A) Central cord syndrome
B) Brown-Sequard syndrome
C) Anterior cord syndrome
D) Complete transverse myelopathy
E) Posterior cord syndrome
Anterior cord syndrome is the lesion of the anterior 2/3 of the medulla spinalis. Proprioception is
characterized by impairment of heat sensation in motor and pain while maintaining. Central cord syndrome
almost always occurs in the cervical region with more pronounced weakness in the arms than the legs and
sacral protection. Hyperextension of the degenerative cervical spine is the most common mechanism of this
injury. Somototropic organism in the spinal cord is the reason for the greater protection of the upper
extremities and the protection of the sacral region. Cervical fibers are located in the innermost and sacral
fibers are outermost. Brown-Sequard syndrome, on the other hand, is paralysis and proprioception under
the lesion level as a result of medulla spinalis hemisection and loss of sensation of pain and heat on the
opposite side. It is usually seen as incomplete; it rarely occurs as described in the books.
Neuroanatomically, the spinothalamic tract, which transmits pain and heat sensation, is described as
crossing the spinal cord, corticospinal and doral colon in the brainstem. Conus medullaris syndrome is
characterized by areflex bladder, bowel and lower extremities due to injury to the sacral cord and lumbar

file:///C|/Users/User/Downloads/small%20intern.htm[06-Aug-19 12:48:59 AM]


nerve roots within the spinal canal. Kauda ekina syndrome occurs with areflex bladder, bowel, and lower
extremities as a result of injury to the lumbosacral nerve roots within the neural canal.
Brown - Sequard syndrome
Anterior cord syndrome
Central cord syndrome
TUSTIME ALL TASS QUESTIONS
21
(Answer A)
SMALL INTERNSHIP
ALL TUSK QUESTIONS. Which of the following brain tumors is not of neuroepithelial origin? (April 2008) A)
Meningioma
B) Oligodendroglioma
C) Medulloblastoma
D) Ependymoblastoma
E) Astrocytoma
Astro cytoma, oligodendroglioma, epandimoma, epandimoblas tom are of neuroectoderm origin.
Medullablastoma is a PNET (primitive neuroectodermal tumor) tumor. It originates from embryonic
neuroectoderm residues. Meningioma is a connective tissue tumor originating from the arachnoid
membrane.
is a tumor carrying. menenjiomaların
They are most common in the 4th and 5th decades, especially at age 45. They are more common in
women. Most
is a common extraaxial brain tumor. They are slow growing mass tumors.
They are encapsulated and mostly benign. 90% of meningiomas are intracranial, of which about 90% are
supratentorial. Origin from arachnoid villi outside the brain
meningothelial cells. Usually unique but Von-Recklinghausen Nörofib-
rheumatism may be multiple in Type-2. It can grow during pregnancy.
Meningioma CT
Treatment: Surgical total excision
(Answer A)
21. Which of the following breathing patterns is bilateral deep hemispheric lesion
Is that indicator? (April 2008)
A) Cheyne-Stokes respiration
B) Central neurogenic hyperventilation
C) Apneustic respiration
D) Biot respiration
E) Sleep apnea
Cheyne Stokes respiration: Decreasing and increasing type of cyclic ventilation, at the time of circulation
characterized by elongation. It occurs in deep hemispheric lesions. Congenital central hypoventilation:
Various changes in the medulla arcuate nucleus
Respiratory patterns
(Answer A)
22. In a patient with myasthenia gravis, which of the following tests
most sensitive to diagnosis? (April 2008)
A) EEG
B) Magnetic resonance imaging
C) Single-fiber electromyography
D) Nerve conduction studies
E) Muscle biopsy
MYASTENIA GRAVIS
Neuromuscular junction disease. Autoimmune post-synaptic acetyl choline receptors developed against an-
There are tikor. Repetitive movements increased muscle weakness and anticholinesterases
improvement is typical.
It can begin at any age. It is common in women aged 20-40 years. 8-10% with thymoma.
The disease usually begins with ocular symptoms and most often with unilateral ptosis. Couple later
sight is added. Patients with these findings alone are called ocular MG (10%).
Difficulty in speaking, difficulty in swallowing, difficulty in chewing and difficulty in breathing are bulbar

file:///C|/Users/User/Downloads/small%20intern.htm[06-Aug-19 12:48:59 AM]


findings.
Although early fatigue may begin earlier in the extremities, bulbar findings
occurs before.
Diagnosis:
Anticholinesterase drugs are used. Edrofonium chloride (Tensilon) is injected. Visible
If there is improvement (improvement in ptosis) the test is positive. EMG is used in diagnosis.
Treatment:
Acetylcholinesterase inhibitors (pyridostigmine), Steroids, plasmapheresis, immunosuppressants,
azothiopyrine,
IgG and thymectomy can be performed.
(Answer C)
Central neurogenic hyperventilation: Anxiety, panic status, sign of pontine lesion
Biot respiration (ataxic respiration): It is seen in serious central nervous system diseases. Bell
sighs, known as contention, are respiratory arrest that eventually comes with apnea.
NEUROLOGY - NEUROSURGERY
TUSTIME ALL TASS QUESTIONS
23
SMALL INTERNSHIP
ALL TUSK QUESTIONS. Which is the most common tumor in posterior mediastinum? (September 2007) A)
Thymoma B) Thyroid tumors
C) Lymphoma D) Teratodermoid tumors
E) Neurogenic tumors
Anterior mediastinal masses
• Thymoma
• Teratoma
• Lymphoma
• Germ cell tumors Middle mediastinal masses
• Pericardial cysts
• Bronchogenic cysts
• Ascending aortic aneurysms Posterior mediastinal masses
• Neurogenic tumors
Neurogenic tumor in posterior mediastinum
(Answer E)
24. Which of the following central nervous system tumors from embryonic residues
does not originate? (September 2007)
A) Craniopharyngioma
B) Chordoma
C) Epidermoid cyst
D) Teratoma
E) Meningioma
DERMOID TUMORS (TERATOM) They are located in the midline or midline of the posterior fossa. The cyst
wall is both epidermal and dermal, such as hair follicles, sweat glands and sebaceous glands.
includes mentees. Dermoid tumors are slowly growing and enlarging completely desquame epithelium,
dressing and sebaceous
becomes filled with material.
These tumors then rupture, causing Aseptic Meningitis, and most patients therefore have recurrent episodes
of bacterial meningitis and the causative microorganism is usually
Stafa. It is epidermitis. Treatment:
Total excision of the tumor. EPIDERMOID CYST
It is benign and originates from embryologically incorrectly located epithelial cells. Keratinous residues fill
the cyst completely.
The pontocerebellar corner is one of the most popular sites for epidermoid cysts.
Epidermoid cyst in the pontocerebellar corner
CRANOPHARYNGEOM Most commonly seen between 5-11 years. (median age 8) Suprasellar developing as
Rathke's scraps
is an ectodermal residual tissue tumor. Short stature, diabetes insipitus, delayed puberty, visual
disturbances (especially bitemporal)

file:///C|/Users/User/Downloads/small%20intern.htm[06-Aug-19 12:48:59 AM]


hemianopsia) and hormone deficiency (especially GH and less ADH)
emerges. It is the most common cause of isolated GH insufficiency due to destructive causes in children. IT

calcification in the sella.


Craniopharyngoma located in the hypothalamus
chordoma
It is a tumor located in the sacrococcygeal region which is the successor of the notochord.
NEUROLOGY - NEUROSURGERY
TUSTIME ALL TASS QUESTIONS
25
SMALL INTERNSHIP
26 TUSTIME ALL TASSION QUESTIONS Sacrococcygeal chordoma (Answer E)
25. Which of the following is not one of the side effects of opiate use? (September 2007)
A) Pruritus B) Respiratory depression
C) Myosis D) Diarrhea
E) Change of mood
Opiate Effects:
• Creates spinal and supraspinal analgesia.
• Develops respiratory depression (KIBAS).
• It is antitussive.
• Causes nausea and vomiting (by stimulating the chemotriger zone).
• Creates myosis
• Hypothermia makes.
• It causes histamine release (hypotension and bronchoconstriction).
• Constipation creates.
• Produces muscle rigidity.
• May cause hyperglycemia by causing catecholamine release.
(Answer D)
26. I received the impression that talking often pauses and has difficulty finding words
In the examination of one patient, it was found that spontaneous speech was normal and understood what
was said except for frequent pauses. Patient showing various to himself
It can describe what objects do, but it is difficult to find their names.
Which of the following is the most likely diagnosis for this patient? (September 2007)
A) Visual agnosia
B) Transcortical aphasia
C) Tactile agnosia
D) Broca's aphasia
E) Anomic aphasia
Aphasia syndromes
Broca (Motor) aphasia: Expressing yourself while understanding what is spoken is quite good (
engine component). Conduction aphasia: In arcuate fascicular lesions connecting the Wernicke and Broca
area
It occurs. Understanding and speaking is normal. But he can't repeat what he heard.
Speech fields
Wernicke (sensori) aphasia: Aphasia type in which the patient's understanding is severely impaired. Fluent
but
There is ridiculous conversation. The condition called verbal paraphasia is seen. For example, when you
want to say ver give the pen 'bilir he can say nerede where is the school'.
Global (total) aphasia: It is seen in lesions involving Broka, Wernicke and Arquat facies. Speech,
understanding and repetition are impaired. It is seen in middle cerebral artery occlusions. In this case
perisilvian region is affected. Anomic aphasia: This is the type of aphasia in which the naming is impaired.
Patients cannot use names. More
Transcortical aphasia: Its main feature is that only its recurrence is preserved.
Subcortical aphasia: It is seen in thalamic and basal ganglion involvement. Classical aphasia syndromes
is a type of atypical aphasia that does not fit.
(Answer E)
27. Unconscious for more than 30 minutes, continuous contractions to the emergency room

file:///C|/Users/User/Downloads/small%20intern.htm[06-Aug-19 12:48:59 AM]


Which of the following is not an appropriate approach for a patient? (September 2007)
A) Ensuring respiration and circulation
B) Rapid initiation of treatment by intravenous injection of lorazepam or diazepam
C) Blood sampling for glucose, BUN, electrolytes, metabolic and toxic scans
D) Slow administration of phenytoin at a loading dose of 15-20 mg / kg less than 50 mg / min
E) 10 mg morphine intramuscularly every 4 hours
Status Epilepticus
• The occurrence of epileptic seizures, without the patient being unconscious, or an epileptic
• The seizure lasts more than 30 minutes.
• 60% of patients are known to be epileptic. The most common cause of anti-epileptic drug
• Sudden discontinuation and infection.
• Mortality in status is up to 10%. The main complications of status are anoxia, hyperthermia,
• Acidosis, electrolyte disorders, DIC, orthopedic damage and behavioral disorders.
NEUROLOGY - NEUROSURGERY
TUSTIME ALL TASS QUESTIONS
27
SMALL INTERNSHIP
28 TUSTIME ALL TUS QUESTIONS Status epilepticus Treatment of status epilepticus:
• Open the airway
• Take blood, add 0.9% NaCl
• 50% dextrose, 50 ml
• Diazepam 10 mg IV
• Phenytoin 18 mg / min infusion
• Intubate
• Phenobarbital 18 mg / min infusion
• Perform general anesthesia
(Answer E)
28. She was examined for sudden onset of severe headache and short-term loss of consciousness.
In a 30-year-old patient, only meningeal irritation findings were detected. Which of the following should be
considered first in this patient? (April 2007)
A) Subarachnoid hemorrhage
B) Brain abscess
C) Meningitis
D) Epidural bleeding
E) Subdural bleeding
(Answer A)
29. What is the first radiological examination after neurological clinical evaluation in a patient with acute
stroke (stroke)?
(April 2007)
A) Computerized brain tomography B) Cerebral angiography
C) Transcranial utrasonography D) Magnetic resonance imaging
E) Direct head radiography
Ischemia is the most common cause of stroke and hypertensive hemorrhage is the second most common
cause of stroke. CT is the first test for stroke patients. It may indicate the earliest bleeding. If there is no
bleeding, infarction is accepted.
Since the edema of infarct will occur within the first 24 hours, CT infection does not show in the acute
period.Answer A)
30. The characteristic type of sensory impairment observed in syringomyelia is
It is si? (April 2007)
A) Hyperactivity in the deep tendon reflexes of the upper extremities
B) Loss of pain-heat sensation without loss of touch sense
C) Loss of touch on both sides
D) Loss of one-sided position sense
E) Cross pain-loss of heat sensation
Central lesion of the spinal cord (syringomyelia syndrome): It is characterized by central cavitation and
gliosis in and around the canal centralis.
Destruction of the central and paracentral region of the spinal cord through several or more segments

file:///C|/Users/User/Downloads/small%20intern.htm[06-Aug-19 12:48:59 AM]


lesions. Acute processes such as contusion due to hemorrhage or trauma, or tumor, syringomyelia.
Causes pathological formations. Syringomyelia, the most common and typical pathological process of this
syndrome
region, more rarely in the lumbosacral region and medulla oblongata (syringobulbi).
Sringomyelia in the cervical spinal cord
In dermatomes (cloaked in both upper extremities for cervical lesion) related to the segmented lesions, loss
of pain and sense of heat (since the fibers crossing that segment are involved)
In myotomes of the segments, weakness and atrophy (due to spinal anterior horn damage) occur in the
form of symmetrical or asymmetric lower motor neuron involvement.
NEUROLOGY - NEUROSURGERY
TUSTIME ALL TASS QUESTIONS
29
SMALL INTERNSHIP
As the lesion enlarges laterally, weakness of the upper motor neuron type, increased tonus and tendon
relexes and pathological reflexes develop in the lower extremities due to the involvement of descending
motor pathways.
There is no deep sensory impairment.
(Answer B)
31. When a sleeping truck driver wakes up with his right leg on the steering wheel, the right
notices weakness in his foot. In the examination of the patient, a low foot on the right, a decrease in the
dorsiflexion of the right foot and stepping on the gait were detected.
In this case, which of the following nerves should be considered damaged? (April 2007)
A) N. tibialis posterior
B) N. femoralis
C) N. peroneus communis
D) N. obturatorius
E) N. femoralis cutaneus lateralis
N.peroneus comminis
innervates muscles. In case of fibula fractures or compression, no-
ropracies. In this case, a low foot occurs. Lumbar 5th nerve
is the root of the nerve.
30 TUSTIME ALL TUS QUESTIONS Fibula fracture and peroneal nerve injury
(Answer C)
32. Sudden and severe headache, short-term loss of consciousness and neck stiffness in a patient with
ptosis, diplopia and mydriasis in the development of the following lesions
which should be considered? (September 2006)
A) Arteriovenous malformation
B) Cerebellar artery aneurysm
C) Anterior communicating artery aneurysm
D) Posterior communicating artery aneurysm
E) Middle cerebral artery aneurysm
Diplopia, mydriasis and strabismus as a result of compression of the third nerve in posterior comminiac
artery artery
It may occur. In addition, SAH is seen in aneurysm rupture.
(Answer D)
33. A 62-year-old male patient complains of headaches that have occurred several times a year for 20
years. Pain in the right eye for about 30 minutes,
The eye feels behind and right at the temple. 4 times a day and once in the same pain once a night.
Which of the following is the most likely diagnosis for this patient? (September 2006)
A) Cluster headache B) Ophthalmoplegic migraine
C) Trigeminal neuralgia D) Brain tumor
E) Painful ophthalmoplegia
CLUSTER HEAD (HISTAMINERGIC HEAD) Serum histamine level is high.
15 minutes or sometimes hours, several times a day or every other day, unilateral perior-
Severe pain attacks with bital localization.
Lacrimation and rhinorrhea can be seen with pain. With 100% oxygen, pain relief is characteristic.
Treatment is the same as migraine.

file:///C|/Users/User/Downloads/small%20intern.htm[06-Aug-19 12:48:59 AM]


In prophylaxis, lithium and corticosteroids are used in addition to the drugs used in migraine prophylaxis.
However, opiate painkillers in patients with acute attack due to histamine liberation
and prophylaxis.
Cluster headache
(Answer A)
34. In the last three months, the level of change in the level of the left eyelid during the day and
Forty-year-old male patient with bilateral vision developed at the end of the study hours, ptosis up to the
upper limit of the left pupil and limitation of the gaze in the left eye
Respectively. Which of the following is the most likely diagnosis for this patient? (September 2006)
A) Left posterior communicating artery aneurysm
B) Left Horner Syndrome
C) Myastenia Graves
D) Tolosa-Hunt Syndrome
E) Cavernous Sinus Thrombosis
NEUROLOGY - NEUROSURGERY
TUSTIME ALL TASS QUESTIONS
31
SMALL INTERNSHIP
MYASTENIA GRAVIS
Neuromuscular junction disease. There is an antibody against autoimmune post-synaptic acetyl choline
receptors. Muscle weakness increased by repetitive movements and improvement of this weakness by
anticholinesterases are typical. It can begin at any age. It is common in women aged 20-40 years. 8-10%
with thymoma. The disease usually begins with ocular symptoms and most often with unilateral ptosis.
Couple later
sight is added. Patients with these findings alone are called ocular MG (10%). Difficulty in speaking,
difficulty in swallowing, difficulty in chewing and difficulty in breathing are bulbar findings. Although early
fatigue may begin earlier in the extremities, bulbar findings
It occurs. Diagnosis: Anticholinesterase drugs are used. Edrofonium chloride (Tensilon) is injected. eye
If there is visible improvement (improvement in ptosis), the test is positive. EMG is used in diagnosis.
Treatment: Acetylcholinesterase inhibitors (pyridostigmine), Steroids, plasmapheresis, immune suppressives,

azothiopyrine, IgG and thymectomy. TOLOSA-HUNT SYNDROME Tolosa-Hunt syndrome presents with
cavernous sinus / superior orbital fissure presenting with painful ophthalmoplegia.
idiopathic granulomatous inflammatory disease. There is involvement of one or more cranial nerves 3, 4, 6,
and 5 of the ophthalmic and maxillary branches of the 5th cranial nerve accompanying unilateral orbital
pain. Diagnosis: Cranial MRI Clinical Course: Tolosa-Hunt syndrome is a rare condition and should be
considered as a first-line diagnosis in patients presenting with unilateral painful ophthalmoplegia, and
responding dramatically to steroid treatment in a short period of time. Treatment: Steroid
(Answer C)
35. Which can be seen several times in a row and intermittently
is a disease characterized by motor or vocal tics? (April 2006)
A) Sydenham's chore B) Tourette's disease
C) Parkinson's disease D) Tardive dyskinesia
E) Huntington's disease
Tourette Syndrome
TS typically begins between the ages of 5-7. Most patients describe tics as unpleasant sensations, such as
tension, or as increased sensitivity to specific parts of the body. Recognizing tics
time is not easy. Tics need to be distinguished from involuntary hyperkinesias. Muscle twitches around the
eyes and fasciculations of the muscle bundles are not teak. In addition, speech obsessions, magnerism
(such as in autism), stereotypes and mental movements in people with mental disorders may be confused.
Tick symptoms vary widely. Tics are often defined as sudden, stereotypic, involuntary movements in
coordinated muscle groups that tend to occur in seizures. The most common tics; excessive blinking (often
associated with short-term blepharospasm), throat clearing, nodding, and facial wrinkles. It occurs mostly in
the upper parts of the body, and is rarely seen in the trunk and lower extremities.
32
TUSTIME ALL TASS QUESTIONS
NEUROLOGY - NEUROSURGERY

file:///C|/Users/User/Downloads/small%20intern.htm[06-Aug-19 12:48:59 AM]


It usually begins as a simple teak from the upper parts of the body, and over time the other parts of the
body
It spreads. Koprolali (socially inappropriate speech, profanity), although dramatic, is not frequent.
The severity and frequency of tics; increases with excitement, tension, boredom.
Treatment of tics:
Treatment of tics is symptomatic, not curative. Neuroleptics are the most effective treatment possible. Used
in TS
neuroleptic doses are not as high as the use for schizophrenia. Usually rule low start, slow
start low, go slow. The best studied drugs are dopamine blocker haloperidol and pimozide. Risperi-
frost is a newly used drug, D2, 5-HT2 and alpha1-adrenergic receptors are effective.
The efficacy of clonidine is controversial. Clonidine skin patches and 0.1 mg tablets are suitable divided
doses -
s can be given.
Unlike postsynaptic dopamine blockers, tetrabenazine reduces presynaptic vesicles.
(Answer B)
36. Amyloid plaques and neurofibrils carrying tau protein in Alzheimer's disease
The brain region in which it is seen is: (April 2006)
A) Hippocampus
B) Cerebellum
C) Hypothalamus
D) Unkus
E) Corpus callosum
Alzheimer's Dementia
• It is the most common cause of dementia in the elderly. dementia is frequently seen.
• There is a 50% prevalence in the population aged 85 years and over.
• Alzheimer's disease is normally found in the neuron membrane in the brain APP (amyloid precursor)
protein (A-beta).
• Beta and gamma secretase enzymes that separate the A-beta fragment from APP
amyloidogenic A-beta (senile plaque).
The disease primarily affects the hippocampus.
• APP inherited on chromosome 21, 20
early dementia develops at age.
• Deposition of tau protein in the formation of microtubules in the axon
neurofibrillary tangles.
• Apo E protein provides stabilization of amyloidotic substance in Alzheimer's. Therefore
Early dementia occurs in homozygous individuals with Apo E4.
• Acetylcholinergic neurons are lost primarily in Alzheimer's disease.
TUSTIME ALL TASS QUESTIONS
33
SMALL INTERNSHIP
34 TUSTIME ALL TUS QUESTIONS A-beta amyloid formation of secretases by cutting the APP in the neuron
membrane
(Answer A)
37. Which of the following is not seen in amyotrophic lateral sclerosis? (April 2006)
A) Slimming of respiratory muscles
B) Dysphagia
C) Sensory defect
D) Fasciculation
E) Hyperreflexia
ALS • Motor neuron disease. It does not hold sensory neurons.
• Goes with both 1st and 2nd motor neuron involvement.
• There is no cognitive, sensory and autonomic dysfunction and (3-4-6. Innovated by the Kronial Nerves)
eye muscles are not involved.
• There is a lack of SOD (superoxide dismutase) in familial ALS forms. symptoms
• Chewing, swallowing, impaired speech, reduced retching reflex and
induced aspiration and tongue fasciculation.
• Weakness of the extremities, spasticis and fasciculations; upper and lower motor neurons being affected

file:///C|/Users/User/Downloads/small%20intern.htm[06-Aug-19 12:48:59 AM]


It reflects. Prognosis: 3-5 years of survival. Death usually results from aspiration and pulmonary infection
It occurs. Treatment
• Riluzole, a NMDA channel blocker, is useful in slowing the course of the disease.
• Ceftriaxone activates the glutamate carriers present in the gliad and causes glutamate in the medium.
and reduce neurotoxicity.
(Answer C)
38. A 60-year-old male patient was brought to the emergency department 2 hours after the onset of acute
right hemiparesis and aphasia. Other than hypertension before the patient's history
learned that he does not have a disease. The patient's blood pressure was 160/85 mmHg.
Which of the following is the most appropriate treatment for this patient who has normal CT results? (April
2006)
A) Intravenous tPA (tissue plasminogen activator)
B) Low molecular weight heparin (LMWH)
C) Warfarin
D) Intravenous heparin
E) Aspirin
Atherosclerotic Thrombotic Occlusive Vascular Disease:
• It is the most common cause of SVO.
• Most commonly affected by arterial cerebri media. Diagnosis: • CT may not show signs of acute infart in
the first 12-24 hours.
• The MR diffusion sequence detects cytotoxic edema within the first hour.
Acute infarction at left ASM feeding site on diffusion MRI
Treatment:
• Patients who come within the first three hours should receive t-PA
• ASA, clopidogrel, heparin
• If KIBAS is present, it should be controlled (mannitol, furosemide, mechanical ventilator)
(Answer A)
39. If pupils are myotic in an unconscious patient,
which should be considered? (April 2006)
A) Brain death
B) Mesencephalic infarction
C) Anticholinergic drug intoxication
D) Opiate intoxication
E) Cerebellar hemorrhage
NEUROLOGY - NEUROSURGERY
TUSTIME ALL TASS QUESTIONS
35
SMALL INTERNSHIP
anisocoria
• Middle brain lesions
• Oculomotor paralysis
• Horner syndrome
Pin point pupilla (Light reflex positive)
• Pons lesions,
• Opiate, organophosphate intoxication
• Myotic eye medications
Bilateral fixed dilated pupilla
• Diffuse cerebral anoxia
• Anticholinergic, sympathomimetic drugs
• Hydrocephalus
(Answer D)
40. Progressive loss of consciousness, right mydriasis and left hemiparesis following head trauma
Which of the following should be considered first?
(September 2005)
A) Falks six herniations
B) Central transtentorial herniation
C) Unkal herniation

file:///C|/Users/User/Downloads/small%20intern.htm[06-Aug-19 12:48:59 AM]


D) Tonsillary herniation
E) Cerebellar herniation
Herniaations in Cyprus
1) Subfalxian Herniation:
Falx is the pushing of the gyrus cinguli under the cerebri.
Headache, nausea, vomiting, vision loss occurs. 2) Transtentorial Herniation:
Uncus and hippocampus from the tentorium is hernia. 3. Cranial nerve remains under compression
(anisocoria), hemiparesis-hemiplegia, deserebre stiffness occurs.
3) Tonsillary Herniation:
From the foramen magnum, they enter the cerebellar tonsils and compress the medulla oblongata.
Sudden loss of consciousness, sudden respiratory loss, sudden intermittant opistotonus. Nuchal stiffness,
loss of retching and cough reflexes are seen.
36
TUSTIME ALL TASS QUESTIONS
Brain Shift types
(Answer C)
NEUROLOGY - NEUROSURGERY
41. A 56-year-old male patient complained of difficulty in walking for the last 3 months
References. Neurological examination revealed decreased sense of position in the left lower extremity
It is found to be present. Peptic ulcer 2 years before the patient's history with a positive Romberg test
It was learned that he had undergone gastrectomy surgery.
Which of the following is the most likely diagnosis for this patient? (September 2005)
A) Narrow spinal canal
B) Intradural-extramedullary tumor
C) Subacute combined degeneration
D) Tabes dorsalis
E) Spinal muscular atrophy
SUBACUTE COMBINED DEGENERATION (NEURO-ANEMIC SYNDROME)
Brain, medulla spinalis, optic nerve and peripheral nerves are affected due to vitamin B12 deficiency.
The most severe damage is seen in the posterior and lateral cords of the medulla spinalis.
Paresthesias, fatigue and weight complaints occur on the extremities. As the disease progresses, spastic-
ataxic gait occurs.
(Answer C)
42. A seventy-year-old male patient presented with a symmetrical quadriparesis that progressed slowly
within 4 days.
Fetching due. The patient's sensory examination was normal and reflexes could not be obtained and
bilateral facial
paresis.
Which of the following is the most likely diagnosis for this patient? (September 2005)
A) Gullian-barre syndrome
B) Cervical medulla compression
C) Pons hematoma
D) Atlanto axial subluxation
E) Basilar artery thrombosis
GUILLAIN-BARRE SYNDROME
Antibodies develop against peripheral nerves. Viral and bacterial (Campilobakter jejuni) with
infections can be triggered.
Initially, sensory symptoms are evident.
Then paralysis begins as distal and progresses ascending.
Diagnosis:
CSF proteins are high in patients. Cells are not seen against high protein in CSF. (Albu-
mino, cytological dissociation)
Treatment:
Plasmapheresis is the first treatment method to be chosen.
In addition, intravenous human immunoglobulin G (IVlG) accelerates recovery in these patients.
(Answer A)
TUSTIME ALL TASS QUESTIONS

file:///C|/Users/User/Downloads/small%20intern.htm[06-Aug-19 12:48:59 AM]


37
SMALL INTERNSHIP
43. Which of the following is not one of the conditions leading to delirium? (September 2005)
A) Cardiac arrhythmia
B) Electrolyte imbalance
C) Posttraumatic stress disorder
D) Hyperparathyroidism
E) Thiamine deficiency
delirium
It occurs when brain functions are affected by any pathology in the body.
Acute onset and often reversible characterized by confusion and impaired consciousness
is a disorder. Cognitive impairment is accompanied by impairment (memory, attention and concentration
ration).
orientation, speech).
Emotional fluctuations, hallucinations, and irrational impulsive violent behavior may occur. Typical
Symptoms exacerbate at night. Almost every clinic is always encountered in daily practice
is a table.
Causes:
Brain-related causes
• Cerebrovascular events
Intracranial infections
• Traumas
• Tumors
• Epilepsy
• Deprivation syndromes
Metabolic
• Liver, kidney failure
• Fluid-electrolyte disorders
• Vitamin deficiencies
• Hypertension
• Hypo-hyperglycemia
• Adrenal pathologies
• Acute pancreatitis
• Hyperthermia
Toxic
• Poisoning
Other
• Surgery
• Arrhythmias
• Trauma
Post Traumatic Stress Disorder
After encountering unusual trauma (earthquake etc.), which can be considered severe for everyone,
relapse, slowdown in reactions, decreased interest in the outside world with certain mental disorder.
38
TUSTIME ALL TASS QUESTIONS
Results:
Anxiety, depression, obsessive recall, compulsive guilt, irritability, bed
wetting and excessive alcohol use. The symptoms of PTSD lasting from 2 days to 1 month are called acute
stress disorder and this condition is considered as trauma.
after 3 weeks.
(Answer C)
44. Which of the following CNS tumors has the worst prognosis? (April 2005)
A) Optical glioma
B) Medulloblastoma
C) Cerebellar astrocytoma
D) Prolactinoma
E) Anaplastic meningioma

file:///C|/Users/User/Downloads/small%20intern.htm[06-Aug-19 12:48:59 AM]


medulloblastoma
• The most common symptom is vomiting and headache.
• In addition, unrest, diplopia, meningismus and cerebellar findings are seen.
• CSF spread through the subarachnoid space is common.
• It is spread by implantation to the places where CSF is spread. Meninges are involved.
• Desmoplastic variant is often found in young adults.
• They are frequently seen in the cerebellum vermis.
Medulloblastoma located in the 4th ventricle on MRI
Treatment: The aim of surgery is aggressive but sensitive resection. They are radiosensitive and 25-30%
are aggressive
radiation therapy. Medulloblastoma is the most malignant tumor of children. When diagnosed, the life
expectancy is up to 9 months.
NEUROLOGY - NEUROSURGERY
(Answer B)
TUSTIME ALL TASS QUESTIONS
39
SMALL INTERNSHIP
ALL TUSKY QUESTIONS. A 50-year-old male patient presented with complaints of increasing hearing loss
and tendency to fall to the right when walking. Time for years from the patient's history
It is learned that when dizziness occurs. Physical examination revealed mild right central facial paresis,
sensory loss in the face, moderate cerebellar tests on the right
disturbance and conductive hearing loss on the right side. Which of the following is the most likely diagnosis
for this patient? (April 2005)
A) Cerebellar bleeding
B) Cerebellar infarction
C) Pontin infarct
D) Cerebellopontin angle tumor
E) Olivopontocerebellar atrophy
The first thing that comes to mind when cerebellopontin angle tumors are mentioned is acoustic neurinoma.
It causes symptoms such as tinnitus, vertigo, and hearing loss. 5th and 7th nerve compression findings if
too large
It may occur.
Right pontocerebellar corner tumor on MRI
(Answer D)
46. In the case of enophthalmus, ptosis and pupillary construc- tion on the left side after surgery in a
patient who underwent neck dissection, firstly
which is considered? (September 2004)
A) Sympathetic nerve trunk trauma
B) Internal carotid artery annealing
C) External carotid artery trauma
D) Reaction to the ointment applied to the eye
E) Lingual artery ligation
Horner in sympathetic nerve trunk trauma or compression tumors (Pancoast lung tumor)
syndrome develops. In Horner syndrome, ptosis occurs when Müller muscle innervation, which is a smooth
muscle that gives tonus to the upper eyelid, is impaired. Since there is also Parasympathetic dominance
The affected side occurs in the eye miosis.
Horner Syndrome
(Answer A)
47. Which of the following is not one of the symptoms of Alzheimer's disease?
(September 2004)
A) Forgetfulness
B) Loss of vibration sense
C) Speech disorder
D) Difficulty in recognizing objects by touch
E) Decrease in motor activity
In Alzheimer's dementia, forgetfulness, aphasia, asterognosia, apraxia, incompetence, decrease in motor
activity, personality change and psychotic symptoms may occur. However, vibration

file:///C|/Users/User/Downloads/small%20intern.htm[06-Aug-19 12:48:59 AM]


deep senses such as the senses are not affected.
(Answer B)
NEUROLOGY - NEUROSURGERY
TUSTIME ALL TASS QUESTIONS
41
SMALL INTERNSHIP
ALL TUSK QUESTIONS. A 25-year-old woman presented with visual loss in her left eye. Physical
examination revealed optic neuritis. Cerebrospinal fluid examination
oligoclonal band was found to be positive, and peri-ventricular hyperintense areas were observed in the
brain on magnetic resonance imaging.
Which of the following is the most likely diagnosis for this patient? (September 2004)
A) Amyotrophic lateral sclerosis
B) Primary lateral sclerosis
C) Myasthenia gravis
D) Multiple sclerosis
E) Tuberous sclerosis
MULTIPL SCLEROSIS
• MS is a demyelinating disease in young adults. symptoms
• Urinary incontinence due to fatigue, paresthesia, retrobulbar neuritis, diplopia and sphincter problems
• Relapses and remissions.
• Myelin basic protein (MBP) is the most important antigen for T cells. MBP-reactive T lymphocytes are
present in the blood and CSF of the patients.
• Serums of MS patients also have antibodies to myelin components (myelin oligodendrocyte glycoprotein-
MOG).
• Optic nerve lesions are among the most common initial findings in MS.
• The first motor neuron, sensory neuron and autonomic CNS neurons, the second motor neuron and
peripheral nerves. Diagnosis
• Presence of plaques on MRI
• Evoked potentials The demonstration of oligoclonal bands in CSF are laboratory findings that support the
diagnosis of multiple sclerosis.
Demyelinating plaques on the periventricular and spinal cord in MS patients
Treatment: Glucocorticoids are used in acute attacks, plasma exchange is done. Again for this purpose MTX,
cyclophosphamide,
IVIG available. INF-Beta is used to maintain remission. T cell with INF-Beta immunomodulatory effect
reduces proliferation and suppresses inflammation. Glatiramer acetate from a random combination of 4
aminoacids (1-lysine, 1-glutamate, 1-alanine, 1-tyrosine)
is the compound formed. It acts by blocking the MHC molecule.
(Answer D)
49. The patient was traumatized four hours ago.
discharge, peripheral facial and abducens paralysis on the same side.
Which of the following is the most likely diagnosis in a patient who is conscious? (April 2004)
A) Intrapontin hemorrhage
B) Wallenberg syndrome
C) Epidural hematoma
D) Subdural hematoma
E) Head base fracture
Petrosis bone fractures may be longitudinal or transverse.
Longitudinal fractures are more frequent, and these fractures are damaged in the middle ear and ossicles.
In transverse fractures, the inner ear is affected and N. facialis is injured.
NEUROLOGY - NEUROSURGERY
Temporal bone longitudinal fracture
Temporal bone transverse fracture
(Answer E)
TUSTIME ALL TASS QUESTIONS
43
50. Which of the following is a sign of cerebellar syndrome? (April 2004)
A) Dysdiadokinesia on the lesion side

file:///C|/Users/User/Downloads/small%20intern.htm[06-Aug-19 12:48:59 AM]


B) Epilepsy
C) Apraxia versus lesion
D) Hypotonia in passive motion
E) Internuclear ophthalmoplegia on the lesion side
The main task of the cerebellum is to obtain and compare information from the brain and muscle,
is to obtain information about. 1- Archicecerebellum: It consists of lobus flocconodularis which is the oldest
part of the cerebellum and
It is about balance. The flocculonodular lobe is associated with the vestibular nuclei and is the first
developed lobe. It's called Archicerebellum. Especially for posture and balance
is necessary. It causes loss of balance and ataxia. 2- Paleocerebellum (anterior cerebellum): It receives
proprioceptive senses from muscles and tendons.
The medulla is associated with spinalis. The lesion has no symptoms. 3- Neocerebellum (posterior
cerebellum): Phylogenetically the newest part. the brain
It linked. Ensures that voluntary movements are coordinated and organized. The width of the movement,
direction, force adjusts. Dysarthria, dysmetria, intense tremor and dysdiadokokinesia occur in
neocerebellum lesions.
Cerebellum lobes
(Answer A)
51. Neurological disease with dementia, usually accompanied by generalized myoclonic contraction
Which of the following? (April 2004)
A) Alzheimer's
B) Creutzfelt jacob
C) Parkinson's
D) Huntington
E) Pick disease
SMALL INTERNSHIP
TUSTIME ALL TUS QUESTIONS 44
Sponging in the brain in Creutzfeldt-Jakob (CJD) disease
(Answer A)
52. Which of the following does not cause mononeuropathy multiplex? (September 2003)
A) DM
B) Cryoglobulinemia
C) PAN
D) Rheumatoid arthritis
E) Tuberculous meningitis
Mononeuritis multiplex is the name given to nerve influences in different plexuses. For example lumbar
low foot in the plexus to show ethylene, brachial hand paresthesia showing the effect of the hand.
Mononeuritis is a typical finding of multiplex vasculitic involvement. Especially in PAN vasculitis
It is typical. In addition, DM can be seen in neuropathy, rheumatoid arthritis and cryoglobulinemic
vasculitis.
(Answer E)
Creutzfeldt-Jakob (CJD) disease
• Creutzfeldt-Jakob (CJD) disease is a rare neurological disorder.
Cerebeller ataxia, myoclonic contractions, dementia and death progresses.
• Typical histopathology shows Spongioform degeneration.
• Vakuoler degeneration is seen especially in cerebral and cerebellar cortex.
• The causative agent was identified in 1982. Prion
is known to be free of any nucleic acid.
• Diseases that replace the normally endogenous prion-associated protein (PrPc) in the human brain
The prion passes through the modified protein (PrPSc). Therefore, neuron connections deteriorate.
• CJD disease occurs. In the last epidemic in the UK, a different form of the disease has emerged and has
developed rapidly (up to a year).
• Veal disease can be transmitted especially in young people fed hamburger-type fastfood.
NEUROLOGY - NEUROSURGERY
TUSTIME ALL TASS QUESTIONS
45
SMALL INTERNSHIP

file:///C|/Users/User/Downloads/small%20intern.htm[06-Aug-19 12:48:59 AM]


53. A 45-year-old male patient complained of headache, which lasted for 16 days and lasted four times a
year for about 14 years, five times daily. Pain in the patient's history
usually occurs at the same time and feels pain around the face on the left side of the face, redness in the
eye,
is said to live. What is the most likely diagnosis for this patient? (September 2003)
A) Episodic tension headache
B) Ophthalmoplegic migraine
C) Episodic cluster headache
D) Cavernous sinus thrombosis
E) Trigeminal neuralgia
(Answer C)
54. A 20-year-old patient presented with progressive hearing loss. On physical examination, sensorineural
hearing loss, mental state deterioration, cerebellar ataxia, deep skin
decrease in frost reflexes, sock-glove style sensory defect and retinitis pigmentosa. Electromyographic
examination shows polyneuropathy.
Which of the following is the most likely diagnosis for this patient? (April 2003)
A) Friedreich's ataxia B) Refsum disease
C) Myotonic dystrophy D) Alport syndrome
E) Paget's disease
REFSUM DISEASE Refsum syndrome is characterized by impaired lipid metabolism in which phytic acid
accumulates in blood and tissues,
is a slow progressive disease characterized by neurological damage. Clinic: There are 4 major findings.
• Retinitis pigmentosa
• Peripheral neuropathy
• Ataxia
• Increased protein in empty fluid (without pleocytosis)
Other:
• Nystagmus
• Anosmi
• Ichthyosis
Etiology of epiphyseal dysplasia:
This syndrome shows an autosomal recessive transition. The main biochemical abnormality is the
accumulation of phytic acid. Patients with refsum syndrome have lost phytic acid degradation. Human body
phanic
can not produce acid, phytic acid is taken through the diet. All symptoms in refsum patients are caused by
this accumulation. Since phytic acid is taken on a diet, foods containing phytic
It prevents the occurrence of symptoms, regression and stabilization of the symptoms that occur. The
defect enzyme in this syndrome is phytoyl, which is found in peroxisomes and cleaves branched fatty acids.

CoA is hydroxylase. (Alpha-oxidase)


46
TUSTIME ALL TASS QUESTIONS
NEUROLOGY - NEUROSURGERY
Epidemiology:
Although this syndrome can occur at any time from early childhood to the age of 50
usually occurs within the first 20 years. Women and men are equally affected.
Treatment:
Refsum disease is the lipid storage disease that responds best to treatment. Phytanic acid containing foods
Removal from the diet lowers the plasma concentration. This decline may not be seen for months. This
This is due to the fact that phytic acid can be stored as phytanate. Depending on diet compliance
As peripheral neuropathy disappears, ichthyosis is reduced. But problems with vision and hearing
not declined. Only progress can be stopped.
Foods containing phytic acid; tuna, cod fish, haddock, lamb, boiled steak, white
bread, white rice, boiled potatoes, egg yolk.
Plasmapheresis may be added to the diet to provide a positive initial response. Early life
The effect of treatment can be increased with standard, timed and correct treatment.
(Answer B)

file:///C|/Users/User/Downloads/small%20intern.htm[06-Aug-19 12:48:59 AM]


55. Which of the brain tumors in adults has the worst prognosis?
(September 2003)
A) Astrocytoma grade III
B) Oligodendroglioma
C) Ependymoma
D) Glioblastoma multiforme
E) Medulloblastoma
Glioblastoma multiforma is the most common brain tumor with the worst prognosis in adults. Children
pilocytic astrocytoma is the most common brain tumor and medulloblastoma is the most common brain
tumor.
(Answer D)
56. For an acute head and neck injury
It should be done? (September 2002)
A) X-ray
B) Keeping the airway open
C) Urinary catheter insertion
D) Intravenous fluid administration
E) Giving fresh blood
Basic life support for trauma A: Airway, B: Breath
A: Circulation. In other words, keeping the airway open and providing ventilation.
(Answer B)
TUSTIME ALL TASS QUESTIONS
47
SMALL INTERNSHIP
57. Which of the following is observed with periorbital ecchymosis in the form of glasses?
(September 2002)
A) Acute subdural hematoma
B) Epidural hematoma
C) Broken head base
D) Intraventricular bleeding
E) Traumatic subarachnoid hemorrhage
Periorbital ecchymosis occurs in skull base fractures.
(Answer C)
58. Which of the following muscles does not develop a defect in myasthenia gravis?
(September 2002)
A) Orbicularis school
B) Levator palpebrae
C) Intercostal muscles
D) Muscles regulating pupillary functions
E) Proximal extremity muscles
Myasthenia gravis develops antibodies to the Nicotinic Ach receptor in the skeletal muscle. Meat in smooth
mus-
no restraint. Therefore, pupil muscles are not affected.
(Answer D)
59. In a diabetic patient, numbness, loss of sensation in the right thumb, index finger and middle finger
the thumb and middle finger can not be touched end to end and thenar atrophy is seen.
Which of the following nerves should be considered in the patient? (September 2002)
A) N.medianus B) N.axillaris
C) N.ulnaris D) N.musculocutaneus
E) N.radialis
In diabetic patients, high blood sugar results in glycation of proteins.
Glycosylated protein products accumulate in tissues and cause microvascular complications.
cause. Neuropathy is the most common chronic complication in DM patients.
Neuropathy can be sensory, motor, autonomic. The most classic neuropathy gloves socks style sense
is a form of neuropathy that goes with the defect. Diabetic foot developing secondary to neuropathy
Complications.
Diabetic foot

file:///C|/Users/User/Downloads/small%20intern.htm[06-Aug-19 12:48:59 AM]


48
TUSTIME ALL TASS QUESTIONS
Diabetic foot
Autonomic neuropathy: Postural hypotension is the main symptom. Impotence occurs in men.
Diabetic gastroparesis due to abnormal gastrointestinal motility may occur. Stomach empties late. also
diarrhea due to bacterial overgrowth (at night) when colon motility decreases.
ebilir.
Diabetic amyotrophy: Atrophy occurs around the femoral muscles due to N. femoralis damage.
Diabetic radiculopathy
Diabetic mononeuropathy: The third most common cranial nerve head pair.
Charcot joint: Articular cartilage is degenerated because of deep sensation.
falls into.
Charcot joint
(Answer A)
60. Which drug is not used in prophylactic treatment of cluster headache?
A) Ergotamine
B) Verapamil
C) Metisergite
D) Propranolol
E) Codeine
In cluster headache, treatment and prophylaxis are the same as in migraine. However, histamine increase in
cluster headache
histamine liberators can not be used. Therefore opiate histamine like codeine lib-
They have no place in the treatment because they are an erator.
(Answer E)
61. Lumbar puncture is contraindicated in which of the following situations?
(April 2002)
A) Spinal cord tumor
B) Intracranial tumor
C) Tuberculous meningitis
D) Myelites
E) Multiple sclerosis
NEUROLOGY - NEUROSURGERY
TUSTIME ALL TASS QUESTIONS
49
SMALL INTERNSHIP
Indications:
• Measurement of CSF pressure
• Cell, chemical and bacteriological examinations (subarachnoid hemorrhages, meningitis, encephalitis)
• Giving radiopaque material for myelography, cisternography
• Contraindications to chemotherapy (intrathecal methotrexate);
• Suspected intracranial mass (risk of cerebral herniation)
• Local infection at the site of LP
• Coagulopathy
(Answer B)
62. The power of speech in a sixty-year-old male patient, half of the right face and body
numbness, lack of force on the right arm. In this patient who is thought to have cerebrovascular event,
what vessel occlusion may have caused the condition? (April 2002)
A) Right carotid artery
B) Left anterior cerebral artery
C) Left middle cerebral artery
D) Right posterior cerebral artery
E) Left posterior inferior cerebellar artery
Atherosclerotic obstruction of the A.serebri media is the most common cause of stroke. Dominance when
ASM clogged
If the (left) hemisphere is affected, aphasia develops. Also occurs motor and sensory loss.
(Answer C)

file:///C|/Users/User/Downloads/small%20intern.htm[06-Aug-19 12:48:59 AM]


63. Head trauma, focal neurological findings, unconscious 62
What is the radiological method required at first admission in an acute stage in a male patient? (April 2002)

A) Anteroposterior head radiograph


B) Ultrasonography
C) Angiography
D) Magnetic resonance imaging
E) Computed tomography
If there are neurological findings in head trauma, if there is loss of consciousness, the first examination is
CT. CT intracranial-
nial compression shows possible bleeding and cranial fracture.
(Answer E)
64. Which of the following is a neuropsychiatric complication related to HIV virus?
It is not? (September 2001)
A) Anxiety B) Depression
C) Dementia D) Trichotillomania
E) Convulsion
50
TUSTIME ALL TASS QUESTIONS
NEUROLOGY - NEUROSURGERY
HIV-related Psychiatric Disorders
Dementia: Causes; HIV and additive CNS infected zions, CNS neoplasms, systemic disorder
CNS and endocrinopathies and CNS side effects of drugs. Delirium: It may occur for similar reasons from
dementia in HIV-infected patients.
Delirium is rarely diagnosed. Anxiety disorders: Patients with HIV infection may have one of the anxiety
disorders.
However, generalized anxiety disorder, posttraumatic stress disorder and obsessive-compulsive disorder are
more common.
Adaptation disorder: Anxious or depressed mood disorder adaptation disorder
5-20% of infections. Depressive disorders: HIV infection reported to meet diagnostic criteria for depressive
disorders
The rate of patients is 4-40%. Substance abuse: Patients regularly suffer from depression or anxiety.
they tend to use drugs. Suicidal ideation and attempts: It may be increased in patients with HIV infection
and AIDS.
Risk factors of suicide; Having friends who died for AIDS-related reasons; positive HIV antibody test result;
relapses, social difficulties related to homosexuality, inadequate social or financial support, dementia or
is the presence of delirium. Extreme anxiety: Although seronegative and non-disease in the high-risk group
people are anxious or have obsessions about virus infection. Symptoms include generalized anxiety, panic
attacks, obsessive-compulsive disorder, and hypochondriasis. Repeated se-
ronegative test results reassure some patients. Supportive or insight-oriented therapy may be appropriate in
those without this confidence.
(Answer D)
65. What is the brain region where intracerebral hemorrhage is most common? (September 2001)
A) Cerebral hemispheres
B) Brain stem
C) Putamen
D) Cerebellum
E) Parietal lobe
Hypertensive Cerebral Hemorrhage Causes of cerebral hemorrhage are hypertensive bleeding, aneurysm or
AVM, bleeding diathesis and
amyloid angiopathy and others. The most common hypertensive hemorrhages are to the level of putamen.
This is because the basal ganglia
of the arteries (a. Talamostriata-a.lentikulostriata) ASM from the right angle. Therefore, systemic pressure is
directly reflected in these vessels.
Pontin bleeding; pinpoint pubil, loss of consciousness and quadriparesis. The aim of all brain hemorrhage is
to control the increase in intracranial pressure.
Treatment:

file:///C|/Users/User/Downloads/small%20intern.htm[06-Aug-19 12:48:59 AM]


• KIBAS measures are taken in case of cerebral hemorrhage.
• Decompression can be applied to superficial white matter hematomas.
Recombinant Factor VII is promising.
TUSTIME ALL TASS QUESTIONS
51
SMALL INTERNSHIP
52 TUSTIME ALL TUS QUESTIONS Right putaminal hypertensive bleeding and midline shift
(Answer C)
66. Which of the following is not a sign of temporal lobe herniation?
(September 2001)
A) Primary optical atrophy
B) Ophthalmoplegia
C) Pyramidal tract involvement
D) Bradycardia
E) Cheyne-stokes respiration
Herniaations in Cyprus
1) Subfalxian Herniation: Gyrus cinguli are pushed to the opposite side under Falx cerebri.
Headache, nausea, vomiting and vision loss occur.
2) Transtentorial Herniation: Uncus and hippocampus from the tentorium are herniated. 3. The cranial
nerve remains under compression (anisocoria), hemiparesis- hemiplegia and deserebre stiffness occur.
3) Tonsillary Herniation: From the foramen magnum, the cerebellar tonsils enter and compress the medulla
oblongata. Sudden loss of consciousness, sudden respiratory loss and sudden intermittant opistotonus.
Nape
hardness, retching and cough reflexes are seen.
Hernia types
(Answer A)
NEUROLOGY - NEUROSURGERY
67. What is the most common spinal cord tumor of intradural extramedullary? (September 2001)
A) Ependymoma
B) Astrocytoma
C) Meningioma
D) Chordoma
E) Angioma
Intradural extramedullary tumors
• Meningiomas (most often)
• Neurinoma (Schwannomas)
Intradural-Intramedullary tumors
• Astrocytomas
• Ependymoma
(Answer C)
68. In a comatose patient, pupils appear as small as the tip of a small needle. This
is bleeding in a patient? (April 2001)
A) Diencephalon
B) Cerebral hemispheres
C) Pons
D) Upper brain stem
E) Bulbus
Pons bleeding, opiate poisoning, organophosphate (cholinergic) drug poisoning
The pin needle consists of the pubil called the head.
(Answer C)
69. If a patient has pseudobulbar paralysis, where is the lesion? (April 2001)
A) Medullary nuclei
B) Pons
C) Cranial nerves
D) Bulber nuclei
E) Tractus corticobulbaris fibers
Pseudobulber palsy

file:///C|/Users/User/Downloads/small%20intern.htm[06-Aug-19 12:48:59 AM]


V, VII, IX, X, XI and XII degeneration of corticobulber pathways leading to the motor nuclei of cranial
nerves
nuna depends. Difficulty swallowing. Speech is peltek and monotone.
The gag reflex is increased and the tongue is immobile, cannot be taken out. Emotional lability and laughter
in patients
and crying crises. The corticobulber path passes through the limbic system. Influence of this path
Because of this, emotional symptoms occur.
TUSTIME ALL TASS QUESTIONS
53
SMALL INTERNSHIP
54
TUSTIME ALL TASS QUESTIONS 4 on MR. Medulloblastoma located in the ventricle
(Answer B)
71. Extension of the sella and suprasellar calcification on head radiography in a young patient
is the most likely diagnosis? (September 2000)
A) Tuberous sclerosis
B) Craniopharyngioma
C) Shopping Mall
D) Sturge-Weber
E) Prolactinoma
Corticobulber Road
(Answer E)
70. Which part of the brain is located in medullablastomas? (April 2001)
A) Corpus collosum
B) Cerebellum
C) Parietal lobe
D) Mesencephalon
E) Tentorium
Pilocytic astrocytoma, medulloblastoma, ependymoma are tumors originating from posterior fossa. Medullo-
blastoma is a PNET tumor originating from the fourth ventricle. Propagation by planting through CSF
typical feature.
NEUROLOGY - NEUROSURGERY
Craniopharyngioma is an embryonic remnant of the anterior pituitary from the Rathke pouch.
is a developing tumor. It causes pituitary insufficiency and diabetes insipitus.
(Answer B)
72. Where do morphological changes occur in Parkinson's disease? (September 2000)
A) Nucleus dentatus B) Cortex
C) Substantia nigra D) Subthalamic nucleus
E) Caudate nucleus
Parkinson's disease is characterized by tremor, rigidity, bradykinesia, and postural instability.
The most common cause is idiopathic Parkinson's disease and it is seen as a basic or additional finding in
other degenerative brain diseases. Other causes for secondary Parkinsonism: drug use
history (such as neuroleptics, reserpine, lithium), intoxications (carbon monoxide, manganese, cyanide,
carbon disulfide, disulfuram), tumor, vascular (multi-infarct), hydrocephalus, hypoxia and trauma.
The characteristic signs of the disease are tremor, rigidity, bradykinesia and posture changes. Parkinson's
tremor is evident at rest and is associated with rough rhythmic, flexion-extension, pronation-supination
movements.
it occurs. Tremor is also common in money-counting style. Tremor proceeds proximally from the distal
extremity, after months or years the other
can Increases emotional stress and carefully severity. Patients have varying degrees of rigidity. Postural
changes may be noticed during the gait examination. Patient general flexion
It is posture. Associated movements of the arms decreased while walking. He walks in small steps, has
difficulty starting to move and ending after starting.
is one of the problems. Signs of facial facial reduction (mask face) are detected. The handwriting is in small
letters and gradually shrinks (micrography) after you start writing.
Loss of inhibitory dopaminergic neurons in the nigrostriatal pathway in physiopathology
It is concerned.

file:///C|/Users/User/Downloads/small%20intern.htm[06-Aug-19 12:48:59 AM]


GABA and Ach release from the striatum increases with loss of dopaminergic neurons. Accordingly, par-
kinson's clinic emerges
Parkinson's Treatment L-Dopa is used in the treatment of Parkinson's. Preventing the destruction of L-dopa
by peripheral decarboxylases
decarboxylase inhibitors are used. These are carbidopa and benzaracid. In addition, anticholinergics,
antiviral agent amantadine, dopamine agonists and an MAO-B inhibitor
selegiline is used.
(Answer C)
73. Where is the infarction seen in the area where a.cerebri media feeds in Broca's aphasia
Seen? (September 2000)
A) Temporal lobe
B) Frontal lobe
C) Fasliculus arcuatus
D) Gyrus angularis
E) Occipital lobe
TUSTIME ALL TASS QUESTIONS
55
SMALL INTERNSHIP
56 TUSTIME ALL KEY QUESTIONS Nourishes the lateral face of the brain. Broca, Wernicke is responsible for
the feeding of the speech and motor and sensory areas of the hand-arm-face.
Middle cerebral artery
Inferior frontal gyrus (pars triangularis): The motor area of speech. In the lesion, Broka (expresive)
aphasia develops, the patient can not speak, but understands well.
Brain gyrus
(Answer B)
74. What is the extramedullary intradural spinal nerve tumor? (April 2000)
A) Meningioma
B) Ependymoma
C) Metastatic tumor
D) Astrocytoma
E) Hemangioblastoma
NEUROLOGY - NEUROSURGERY
Intradural, extramedullary tumors
• Neurofibromas
• Menengiom
• Exotic ependymoma
Intradural, intramedullary tumors
• Astrocytoma
• Ependymoma
Extradural tumors
• Usually metastatic tumors
(Answer A)
75. Which of the following conditions does not have papilla stasis? (April 2000)
A) Malignant phase of arterial hypertension
B) Increased intracranial pressure syndrome
C) Malignant thyrotoxic exophthalmitis
D) Central retinal vein occlusion
E) Amyotrophic lateral sclerosis
Papilledema has increased intracranial pressure (bleeding, tumor, meningitis, hydrocephalus, caranial
synostosis ..)
It is a manifestation of the optic nerve in the eye with clearing the boundaries of the optic nerve. Pap-
iledema may cause optic atrophy when it lasts long. Papilledema is also a retinal vessel
disease, hypertension, vasculitis and thyroid ophthalmopathy.
ALS is a motor neuron disease. Papilledema is not seen.
(Answer E)
76. What is the most common cause of spontaneous intracerebral hemorrhage? (April 2000)
A) Aneurysm

file:///C|/Users/User/Downloads/small%20intern.htm[06-Aug-19 12:48:59 AM]


B) Hypertension
C) Tumor
D) AV malformation
E) Blood dyscrasias
Hypertension is the most common cause of spontaneous intracerebral hemorrhage. Most common putaminal
in hypertension
bleeding.
Subarachnoid hemorrhage and parenchymal hemorrhage may occur in AV malformation. Willis polygon
Subarachnoid hemorrhage is seen in Berry aneurysm.
(Answer B)
TUSTIME ALL TASS QUESTIONS
57
SMALL INTERNSHIP
77. Primidone is a special medicine for which of the following diseases? (April 93)
A) Schizophrenia
B) Neurosis
C) Depression
D) Epilepsy
E) Korea
Primidone is an anti-epileptic drug that acts on the return of phenobarbital. Also in essential tremor
used.
(Answer D)
78. Which does not support the diagnosis of brain death? (September 99)
A) Increased deep tendon reflexes
B) Deserebration rigidity
C) Heart arrest
D) No spontaneous breathing
E) Straight drawing of EEG
CLINICAL DEATH
It is characterized by respiratory and cardiac arrest.
CEREBRAL (CORTICAL) DEATH
It is characterized by death of cerebral hemispheres, cortex, spontaneous breathing despite deep coma
would.
BRAIN STEM DEATH
It is characterized by loss of all brain stem reflexes, including spontaneous breathing. Medical and legal
authorities,
definitive measurement of brain stem death even if the heart continues to beat when supported by
mechanical ventilation
accepts.
BRAINDEATH
All brain tissue; cerebrum, cerebellum, midbrain and brain stem death.
BIOLOGICAL DEATH
It is the death of brain, heart, lungs, kidneys, liver, skin, etc. of all tissues. If not resuscitated
biological death following clinical death is inevitable.
DETERMINATION OF BRAIN DEATH
The degree of coma can be determined according to the Glasgow Coma Scale (GCS) (Table I). Accordingly
GKS: 13-15
minimum, GCS: 9-12 moderate, GCS≤8 severe brain injury and GCS <5 deep coma.
The diagnosis of brain death is clinically confirmed by various supporting tests and radionuclide imaging
methods.
It can be put.
58
TUSTIME ALL TASS QUESTIONS
NEUROLOGY - NEUROSURGERY
Glasgow Coma Scale
Eye opening score
Spontaneous → 4 Commandments → 3 Painful stimuli → 2 None → 1 Motor response

file:///C|/Users/User/Downloads/small%20intern.htm[06-Aug-19 12:48:59 AM]


Limb movements normal by order → 6 Painful stimulant localization → 5 Painful stimulus reaction → 4
Painful stimulus flexion → 3 Painful stimulus extension → 2 No response to pain → 1 Oral answer
Oryante-noun, age → 5 Confusion → 4 Improper speech → 3 Pointless grunt → 2 No answer → 1
CLINICAL FINDINGS SUPPORTING TESTS 1. EEG; only gives information about the brain cortex 2.
Brainstem Evoked Potential (BAEP); It shows the electrophysiological activity of the brain death region. It is
the most appropriate test that shows brain stem death, which can be performed at the bedside and does
not harm the patient, and is sufficient to confirm the clinical diagnosis. The test is not performed if one of
the following:
• Pathology of Kohlear transmission path
• VIII. Cranial nerve incision
• Pons hemorrhage
• Posterior fossa operation 3. Computed Tomography: The presence of the cerebral vasculature indicates
the presence of cerebral circulation.
MRI (Magnetic Resonance Imaging) angiography: Cerebral circulation is very good, but the ventilator
There is difficulty in application in cases. Transcranial Doppler Sonography (TDS): It provides information
about carotid arteries and basilic arteries. Radionuclide Techniques: Precisely shows the cerebral circulation,
the application is easy. Transplantation
shows the blood to be used for the organs. SPECT (Single Photon Emission Tomography): Brain perfusion is
investigated with radioactive material (Tc99m HMPAQ). The absence of radioactive material uptake in the
brain tissue indicates that there is no brain perfusion and makes a definitive diagnosis of brain death.
(Answer A)
TUSTIME ALL TASS QUESTIONS
59
SMALL INTERNSHIP
79. The most common localization of intracranial hemorrhage in patients with hypertension
Where is it? (September 99)
A) Talamus B) Putamen
C) Cerebellum D) Brain stem
E) Cerebral cortex
The most common bleeding occurs in patients with hypertension to the basal ganglia (putamen). Basal
ganglia
hemorrhage as a result of compression of the capsule interna and hemiplegia, hemihipoesthesia occurs. In
addition, intracranial pressure and herniation may occur as a result of bleeding.
(Answer B)
80. Which region of the central nervous system has imbalance in the lesion? (April 99)
A) Anterior hypothalamus B) Cerebellum anterior lobe
C) Cerebellum posterior lobe D) Cerebellum flocculonodular lobe
E) Occipital cortex
45. See the description of the problem.
(Answer D)
81. Most commonly in the 3rd ventricle, cerebellum and optic nerve in children and adolescents
Which glial tumor is seen? (April 99)
A) Glioblastoma multiforme B) Pilocytic astrocytoma
C) Ependymoma D) Anaplastic astrocytoma
E) Oligodendroglioma
Pilocytic Astrocytomas
It is the most common brain tumor in the pediatric age group. It occurs in children and adolescents and
typically occurs in one of three localizations and is benign.
Approximately 2/3 of the cerebellum, 25-30% in the optic nerve and chiasma regions, 10-15%
settles in the cerebral hemisphere.
These tumors, which are usually cystic, have a mural nodule consisting of solid tumor tissue. histopatho-
Rosenthal fibrils are seen in theology.
Treatment: Surgery, radiotherapy, chemotherapy
60
TUSTIME ALL TASS QUESTIONS
(Answer B)
NEUROLOGY - NEUROSURGERY

file:///C|/Users/User/Downloads/small%20intern.htm[06-Aug-19 12:48:59 AM]


82. The most probable diagnosis in a female patient with ptosis whose symptoms improved after
administration of edrofonium
Which is? (April 99)
A) Multiple sclerosis
B) Myasthenia gravis
C) Central facial paralysis
D) Oculomotor paralysis
E) Horner syndrome
MYASTENIA GRAVIS Neuromuscular junction disease.
There are antibodies developed against autoimmune post-synaptic acetyl choline receptors. Repetitive
movements
increased muscle weakness and improvement of this weakness with anticholinesterases are typical.
It can begin at any age. It is common in women aged 20-40 years. 8-10% with thymoma.
The disease usually begins with ocular symptoms and most often with unilateral ptosis. Couple later
sight is added. Patients with these findings alone are called ocular MG (10%).
Difficulty in speaking, difficulty in swallowing, difficulty in chewing and difficulty in breathing are bulbar
findings.
Although early fatigue may begin earlier in the extremities, bulbar findings
It occurs.
Treatment
• Acetylcholinesterase inhibitors (pyridostigmine),
• Steroids, plasmapheresis, immune suppressives, azothiopyrine, IgG and thymectomy can be performed.
Diagnosis • Anticholinesterase drugs are used. Edrofonium chloride (Tensilon) is injected.
• The test is positive if there is visible improvement (improvement in ptosis).
• EMG is used in diagnosis.
(Answer B)
83. A 60-year-old male patient who suddenly came to be absurd
What do you think of the patient if the recurrence is impaired? (April 99)
A) Alzheimer's aphasia B) Global aphasia
C) Broka aphasia D) Wernicke aphasia
E) Conduction aphasia
Wernicke conversation area
• Function: The sensory center of speech.
• Lesion: Wernicke (receptive) aphasia develops. The patient does not even understand that he / she
speaks, speaks fluently and generally does not have hemiparesis, confuses with psychosis.
Types of aphasia
• Motor aphasia (Broca's aphasia): The patient understands but cannot speak.
• Sensorial (Wernicke aphasia): The patient does not understand what is spoken but speaks fluently.
• Conductive aphasia: The patient understands, but cannot repeat.
• Global aphasia: The most common type of aphasia. The patient neither understands nor talks.
TUSTIME ALL TASS QUESTIONS
61
SMALL INTERNSHIP
62 TUSTIME ALL KEY QUESTIONS Cyrus and their functions (Answer D)
84. Which of the following is in the perisilvian space due to middle cerebral artery occlusion
commonly seen as a result of necrosis? (April 99)
A) Wernicke aphasia
B) Broka aphasia
C) Global aphasia
D) Literal paraphasia
E) Conduction aphasia
Motor speech (inferior frontal gyrus-pars triangularis) Function: The motor field of speech,
Lesion: Broca (expresive) aphasia develops, the patient can not speak, but understands well.
(Answer C)
85. Which of the following results from bilateral coronal suture synostosis? (April 99)
A) Scaphosephaly
B) Trigonosephal

file:///C|/Users/User/Downloads/small%20intern.htm[06-Aug-19 12:48:59 AM]


C) Brachycephaly
D) Oxycephaly
E) Plagiocephaly
Craniosynostoses
Craniocinostosis is the early closure of cranial sutures. Deformities in the head due to premature closing
suture
interest. Primary craniocinostosis is classified according to the affected calvary suture.
Sagittal synostosis is the most common type of craniocinostosis and usually occurs as an isolated finding.
Early sagittal suture
closure results in the appearance of a long and narrow head called scaphosephaly and dolicosephaly.
the Skafosefal
NEUROLOGY - NEUROSURGERY
The second frequency is early closure of the coronal suture. Short and wide head appearance in bi-lateral
coronal synostosis
brachycephaly, which occurs. Unilateral coronal synostosis is called asymmetric plagiocephaly
The head shape is composed. Oxycephaly or laparotomy in early closure of lambdoid suture
occurs appearance called risefali. The appearance of trigonosephaly occurs in metopic craniocinostosis.
brachycephalic
(Answer C)
86. What is the radiological finding of intracranial pressure increase in adults? (April 99)
A) Opening of sutures
B) Head growth
C) Sellada erosion
D) Increased convolutional symptoms
E) Puffiness in the Fontanel
In the pediatric age, when intracranial pressure increases, there is growth around the head, suture opening,
swelling of the fontenel and eye deviation in the setting sun. Sella floor erosion in pituitary tumors
and adult-type intracranial pressure increase.
(Answer C)
87. Which of the following head pairs is affected
is the reason for seeing? (September 98)
A) 3rd cranial nerve
B) 8th cranial nerve
C) 6th cranial nerve
D) 1st cranial nerve
E) 7th cranial nerve
The nerve that has the longest course in the head is the 6th nerve. Therefore, the most common cases of
intracranial pressure increase
is the affected nerve. Strabismus develops in paralysis.
(Answer C)
88. Which of the following is not extrapramidal system finding? (April 99)
A) Korea
B) Atherosis
C) Asterixis
D) Hemiballismus
E) Grandmal epilepsy
TUSTIME ALL TASS QUESTIONS
63
SMALL INTERNSHIP
64 TUSTIME ALL KEY QUESTIONS BASAL GANGLIONS Basal ganglia are particularly responsible for
performing rhythmic movement (such as handwriting).
The nucleus consists of caudatus, putamen, globus pallidus, nucleus subthalamicus and substantia nigra.
The main function of the basal ganglia is through the fibers passing through the thalamus, affecting the
motor cortex.
Is MEK. The aim of the basal ganglia is to plan and coordinate movement. It is also responsible for the
affective and cognitive functions of the basal ganglia.
Basal ganglia

file:///C|/Users/User/Downloads/small%20intern.htm[06-Aug-19 12:48:59 AM]


The structure formed by nucleus caudatus and putamen together is called striatum. Basal ganglia help the
functions of the motor centers. Basal ganglia input Striatum, output substansia
nigra and globus pallidus. Ballismus occurs in the subthalamic nucleus lesion. If unilateral, it is on the
opposite side of the lesion and
this is called hemiballismus, involuntary contractions. Atletosis occurs in the corpus striatum lesion. Dystonia
and Huntington's chorea
They are disorders. Huntington's chorea contains cholinergic striatum and degeneration of GABA erergic
fibers.
Substantia nigra lesions cause parkinsone. Dopaminergic fibers go into the striatum from substantia nigra.
There is degeneration in the nigrostriatal pathway in Parkinson's disease.
Parkinson's disease is characterized by rigidity, hypokinesia, and tremor.
(Answer E)
89. A 65-year-old chronic alcoholic patient had unconsciousness, ataxia, nystagmus and ophthalmology.
moplegia is the most likely diagnosis? (September 98)
A) Delirium tremens
B) Acute alcohol intoxication
C) Wernicke's encephalopathy
D) Cerebellar degeneration
E) Hypoglycemia
Wernicke's Encephalopathy and Korsakoff Psychosis
later, it was described as Wernicke-Korsakoff Syndrome (WKS) because of its common etiologic, clinical and
histopathological features.
NEUROLOGY - NEUROSURGERY
Wernicke's encephalopathy (polioencephalitis hemorrhagica superioris) is characterized by acute or subacute
ocular findings, ataxia and confusion. In Korsakoff Psychosis, while other cognitive functions are relatively
preserved, significant impairment is in the memory area. Korsakoff Psychosis is frequently seen with
Wernicke's Encephalopathy. It would not be wrong to define Korsakoff Syndrome as psychic symptoms
added during the course of Wernicke's Encephalopathy. Avitaminosis and especially thiamine (B1) deficiency
are responsible for the etiology. Autopsy studies showed a selective decrease in the activity of thiamine-
dependent enzymes. In addition, WKS findings were observed in animals fed B1 poor diet. Syndrome is
thought to occur due to insufficient dietary intake of thiamine. The most frequent follow-up of this
syndrome in chronic alcoholics has led to the suspicion that the direct neurotoxic effect of alcohol may be
responsible for the pathogenesis along with the limitation of thiamine intake. Clinical signs and symptoms
The most common symptoms of the syndrome are impaired mental status, oculomotor findings and
unstable
It is walking. Mental Findings: Confusion is the most common mental symptom in acute WKS syndrome.
The patient was apathic and lost orientation. Ataxia and oculomotor findings accompany the confusional
state. If treatment is not started during this period, the disease progresses and persistent symptoms occur.
Confusional state turns into serious amnesia. Anterograde amnesia: In this period, while no pathological
findings related to other cognitive areas of the patient are encountered, serious impairment in near memory
manifests itself as the failure to learn new verbal and non-verbal information, not remembering recent
events, not being able to remember the names and faces of new people, and not remembering current
events. Failure to learn new information is the result of the encoding defect in the first step of the
recording process of the stimulus.
Apart from memory problems, sometimes frontal findings such as problem solving and concept formation
difficulties can be found. Another cause of memory impairment is the effect of the frontal system on
retrieving information stored in memory functions. Intellectual abilities are normal except for amnesia.
Retrograde amnesia: WKS cases cannot remember social events and information about themselves, which
are in a period extending back from the onset of the disease. The further backward the disease is, the
better the information of that period is remembered. In other words, the patient cannot recall his childhood
and early adulthood, but cannot remember his recent history from the onset of the disease. The severity of
retrograde amnesia was found to be directly proportional to failure of tests showing frontal lobe function.
The frontal lobe plays an important role in planning and initiating systematic memory research. Due to this
disorder, information learned as premorbid cannot be recalled and retrograde amnesia occurs. Ophthalmic
findings: Nystagmus is the most common finding. Horizontal nystagmus is often seen in the direction of
view. In some cases, vertical or rotational nystagmus may also be observed. The limitation of external eye
movements first manifests itself in horizontal eye movements. Subsequently, the limitation of vertical eye

file:///C|/Users/User/Downloads/small%20intern.htm[06-Aug-19 12:48:59 AM]


movements is added to the table. Restriction in eye movements can be observed in asymmetric bilateral
lateral rectus paralysis and conjugate gaze restriction, which may also be seen in vertical gaze, from total
ophthalmoplegia. Decrease in pupillary light response can be seen, but total reflex can not be obtained is
not a detectable finding. A mild ptosis may accompany the clinical picture.
TUSTIME ALL TASS QUESTIONS
65
Ataxia:
There is trunk ataxia in the foreground. The patient walks unevenly, wobbling, opening his steps.
This may be accompanied by extremity ataxia. In addition to the involvement of the cerebellum, vestibular
system
is responsible for the findings.
Diagnosis
Good anamnesis, strict diet, alcohol consumption and consumption
questioning is required
(Answer C)
90. Young female patient with left hemiplegia is brought to the emergency room. The patient who denied
paralysis
If there is no response to visual stimuli on the left side, the most likely location of the lesion in this patient
Which is? (September 98)
A) Right frontal lobe
B) Right parietal lobe
C) Left frontal lobe
D) Right occipital lobe
E) Right capsule interna
Neglect occurs in parityal lobe lesions in the non-dominant hemisphere (right hemisphere). that
the reason is that the right parietal hemisphere provides 3-dimensional spatial conformation.
(Answer B)
91. Which of the following symptoms is not seen in Sturge-Weber syndrome? (September 98)
A) Choroidal hamartoma
B) Glaucoma
C) Angiomas
D) Nevus flammeus
E) Convulsion
SMALL INTERNSHIP
TUSTIME ALL TASS QUESTIONS 66
Struge - Weber
NEUROLOGY - NEUROSURGERY
STRUGE-WEBER DISEASE
Facial nevus (port wine color), seizures, hemiparesis, intracranial calcifications and mental retar-
is a disease characterized by dation. 1 / 50.000, sporadic seen Facial nevus from birth, unilaterally holds the
upper part of the face and eyelid. Same side
buphthalmos and glaucoma in the eye. Seizures develop within the first year. Typically resistant to focal
tonic-clonic anticonvulsants in the contralateral body of the facial nevus
seizures. Progressive hemiparesis develops. In 50% of patients, cerebral atrophy secondary to seizures and
mental retar-
dation develops. Disorder and calcification of leptomeningeal vascularization of the brain, especially in the
occipital region
CT and cranial radiography. Treatment is aimed at preventing seizures. Children whose seizures can be
controlled show normal development.
(Answer A)
92. Which of the following increases brain blood flow? (April 98)
A) CO2 low B) O2 height
C) Hyportermia D) Hypotension
E) Low pH
Conditions such as acidosis and increase in PCO2 cause vasodilatation in the blood vessels of the brain.
formation. The most important factor affecting the brain vessels is PCO2.
(Answer E)

file:///C|/Users/User/Downloads/small%20intern.htm[06-Aug-19 12:48:59 AM]


93. Which is not a feature of Alzheimer's? (April 98)
A) Common in 65-70 years
B) Memory impairment
C) There is a detection flaw
D) Disorientation may occur
E) Depressive temperament and paranoid personality changes can be seen.
Alzheimer's disease is a degenerative disease with amyloid deposition in the brain. It progresses with
memory impairment. Cognitive abilities such as location, time and person orientation are disrupted. In
addition, personality changes and mood disorders may occur in patients.
(Answer C)
94. Which of the following is seen in sixth cranial nerve paralysis? (April 98)
A) There is complete immobility
B) Eye cannot look outward
C) Eye cannot look outward
D) Eye cannot look outward
E) Eye cannot look inward and strabismus is seen
(Answer D)
TUSTIME ALL TASS QUESTIONS
67
SMALL INTERNSHIP
95. Where is the lesion in bitemporal hemianopia? (April 98)
A) Geniculate ganglion
B) Calcaneal sulcus
C) Optical chiasm
D) Occipital cortex
E) Fovea centralis
VISION WAYS
The field that both eyes can see is called the visual field. Fibers from the nasal halves of the retina of both
eyes cross the midline. This cross
The vine is called opticum. The visual pathway after Chiasma opticum is tractus opticus. One side is in the
tractus opticus;
fibers from the temporal half of the retina of the same eye and fibers from the nasal half of the retina of
the opposite eye.
Most of the fibers that make up the Tractus opticus synapse with third neurons in the corpus geniculatum
lateral (CGL). For some reflexes related to fiber visual stimuli, without undergoing CGL
area goes to pretectalis and colliculus superior.
The third neuron extensions starting from CGL go to the primary visual cortex (Brodmann area 17), called
tractus geniculocalcarinus (radiatio optica). The major artery of the visual cortex, a.
cerebri posterior. LESIONS OF VISUAL WAYS
Lesions in front of the chiasma opticum only in the eye on the lesion side, chiasma opticum et al.
The lesions of the structures in the eye cause loss of visual field in both eyes.
Anopia is loss of vision. Incomplete visual field defects are called scotomas. Sagittal destruction of Chiasma
opticum (such as adenohypophysis tumors); both eyes
retinal fibers from the nasal side of the bitemporal hemianopia is involved.
68
TUSTIME ALL TASS QUESTIONS
Ways of seeing
(Answer C)
96. Chronic, non-episodic vertigo, resonance, hearing loss V, VII and VIII
If the paralysis of the lesion, where can the lesion be? (September 97)
A) Capsula interna
B) Spinal cord
C) Cerebellopontin corner
D) Temporal lobe
E) Medial longitidunal fasciculus
In pontocerebellar corner tumors, compression-related function losses occur in pairs of 6, 7, 8 fafa from this
corner. The most common corner tumor is acoustic neurinoma. Vertigo, tinnitus, hearing

file:///C|/Users/User/Downloads/small%20intern.htm[06-Aug-19 12:48:59 AM]


loss is a classic symptom of acoustic neurinoma.
(Answer C)
97. Which is the most sensitive test in multiple sclerosis? (September 97)
A) CT
B) Angiography
C) MRI
D) 2 way head radiograph
E) Brain scintigraphy
In multiple sclerosis, demyelinating plaques form in the periventricular white matter. These plates are T2-
MR
Contrast-retaining MS plaques are considered active (persistent inflammation).
Periventricular demyelinating plaques in MS
(Answer C)
98. Bilateral hemiparesis, speech, chewing, swallowing, pharynx and tongue movements-
What is the most likely diagnosis in a patient with weakness, laughter and crying attacks?
(September 97)
A) Pseudotumor cerebri
B) Pseudobulbar paralysis
C) Facial paralysis
D) Parkinson's
E) Pseudohypertrophic muscular dystrophy
NEUROLOGY - NEUROSURGERY
TUSTIME ALL TASS QUESTIONS
69
SMALL INTERNSHIP
Pseudobulber palsy
V, VII, IX, X, XI and XII degeneration of corticobulber pathways leading to the motor nuclei of cranial
nerves
nuna depends.
Difficulty swallowing. Speech is peltek and monotone.
The gag reflex is increased and the tongue is immobile, cannot be taken out. Emotional lability and laughter
in patients
and crying crises.
(Answer B)
99. Which of the traumatic head fractures develop as a result of injury to the middle meningeal artery?
(September 97)
A) Subarachnoid hemorrhage
B) Subdural bleeding
C) Epidural bleeding
D) Bleeding to the base of the pons
E) Hematoma in external capsule
Epidural Hemorrhage
• It often accompanies fractures in the head bones.
• It is due to A.meningia media bleeding.
• CT appears as lens-shaped (biconvex).
• In 20% of cases, it forms a lucid interval. 2-3 hours after head trauma
With the sudden closure of consciousness can create an outgoing table.
Diagnosis: CT
Treatment: Surgically evacuated
(Answer C)
one hundred. Which of the following is the acute phase of dorsal medulla spinalis injury?
not seen? (April 97)
A) Urinary retention
B) Flask paraplegia
C) Increased patella and achilles reflexes
D) Loss of sweating in lower extremity
E) Sensory loss in the lower extremity

file:///C|/Users/User/Downloads/small%20intern.htm[06-Aug-19 12:48:59 AM]


In spinal cord injuries, spinal shock develops below the level of injury. This is 3-4 weeks
Reflections disappear during the period. Then the reflexes come back. Cortical inhibition is removed
long-term reflexes are taken alive-increased.
(Answer C)
70
TUSTIME ALL TASS QUESTIONS
101. Which of the following nerves innervate the forearm, wrist, and finger extensor muscles and become
stuck as a result of incision, bullet wound or hard external surfaces
Creates “low hand”? (April 97)
A) N. medianus
B) N. radialis
C) N. ulnaris
D) N. axillaris
E) N. musculocutaneus
N.radialis, a.profunda brachi together with the humerus diaphysis. Press on the humerus or
radial nerve damage may occur in humeral shaft fractures. The radial nerve innervates the forearm extensor
muscles. Therefore, low hand occurs in patients.
Radial nerve tracing
(Answer B)
102. Which of the following tumors is more common in the posterior mediastinum? (September 96)
A) Thymoma
B) Thyroid tumors
C) Neurogenic tumors
D) Teratoma
E) Bronchogenic cyst
Anterior mediastinal neoplasms: Teratoma, T cell lymphoma, thymoma Posterior mediastinal neoplasms:
Neurogenic Tumors
NEUROLOGY - NEUROSURGERY
TUSTIME ALL TASS QUESTIONS
71
SMALL INTERNSHIP
72 TUSTIME ALL TUS QUESTIONS Mediastinal neoplasms (Answer C)
103. The most common recurrent meningitis factor in rhinorrhea after head trauma
Which is? (September 96)
A) S. aureus
B) N. meningitidis
C) S. pneumonia
D) Listeria monocytogenes
E) E. coli
Streptococcus pneumonia is the most common cause of meningitis after head trauma. Penetrating head
trauma
then the most common meningitis agent is Staphylococcus aureus.
(Answer C)
104. Which tumor is most likely to cause acromegaly? (September 96)
A) Somatotropic adenoma
B) Craniopharyngioma
C) Acidophil adenoma
D) Basophilic adenoma
E) Lactotropic adenoma
Acromegaly is caused by tumors originating from somatotropic cells secreting GH. ACROMEGALY
• Acidophilic (or chromophobic) pituitary adenoma
• In the age group of 30-50
• Can be seen in any age group
• It is the most common pituitary tumor when diagnosed.
Growth Hormone may also be due to ectopic release. (eg Pancreatic tumors)
• Headache is the most common symptom.
• Increased risk of colon Ca and polyp.

file:///C|/Users/User/Downloads/small%20intern.htm[06-Aug-19 12:48:59 AM]


• Prolactin is most commonly secreted with GH.
Clinical findings of acromegaly
Clinic:
• Bone and soft tissue changes
• Growth at the ends
• Soft tissue growth
• Dislocated forehead - prognathism
• Visceromegaly
• Proximal muscle weakness (Myopathy)
• Gynecomastia
• Thickening of skin folds
• Interstitial edema
• Osteoarthritis
• Pressure neuropathies; The most common bitemporal hemianopsy
Metabolic changes
• Polydipsia
• Dehydration secondary to hyperhidrosis
• Glucose intolerance - osmotic diuresis
• Galactorrhea (prolactin release)
• Hypertension
• Hyperphosphatemia (tubular phosphate reabsorption)
• Increased sweating and sebaceous gland function
• Hypercalcuria, kidney stones
NEUROLOGY - NEUROSURGERY
TUSTIME ALL TASS QUESTIONS
73
SMALL INTERNSHIP
System Symptoms Findings
General weakness, sweating, hot intolerance,
weight increase
Skin and subcutaneous tissue
74 TUSTIME ALL TUSK QUESTIONS Growth in hands and feet, coarse facial features, oily skin, excessive
hair
Moist skin, skin folds, acanthosis nigricans, increase in foot size
Headache Parotid growth, frontal
dish
Eyes Visual impairment Visual visual defects
Nasal, throat, paranasal sinuses Sinus congestion, voice change,
the malocclusion
Tongue growth, prognathism, tongue marks, opening between teeth
Neck Goiter, sleep apnea
Cardiorespiratory Congestive heart failure Hypertension, cardiomegaly
Genitourinar system Libido reduction, oligomenorrhea, im-
potency, infertility, kidney stone
Neurological system Paresthesia Carpal tunnel syndrome
hypersomnoloscence
Muscles Weakness Proximal myopathy
Skeletal system Joint pain Osteoarthritis
Diagnosis: • Oral glucose suppression test: Plasma GH suppression after oral glucose uptake
absence, sometimes paradox increase (the easiest and specific test)
• Morning fasting GH measurement> 10 ng / ml
• Circadian rhythm loss in GH
Radiology: MR
Treatment: Transsphenoidal pituitary (first choice) - Radiotherapy Octreotide (Somatostatin synthetic
analogue): Side effects DM, malabsorption, gallstones
(Answer A)
NEUROLOGY - NEUROSURGERY

file:///C|/Users/User/Downloads/small%20intern.htm[06-Aug-19 12:48:59 AM]


105. What is the drug of choice for trigeminal neuralgia? (September 96)
A) Phenobarbital B) Aspirin
C) Steroid D) Carbamazepine
E) Phenytoin
(Answer D)
106. Right peripheral facial paralysis, right 6th cranial nerve paralysis and left hemiparesis
If so, where is the lesion? (September 96)
A) Left pons
B) Right capsule interna
C) Left capsule interna
D) Right pons
E) Left thalamus
Pons lesions show loss of function in 5, 6, 7, 8 head pairs. On the opposite side of the body
Sensory and motor loss occurs.
(Answer D)
107. What is the most common localization of brain tumor in children? (April 96)
A) Supratentorial
B) Infratentorial
C) Pineal
D) Pituitary
E) Middle brain
Brain tumors in children are located in infrataentorial locations, whereas brain tumors in adults are located
in supratentorial locations. The most common pediatric brain tumor is pilocytic cystic astrocytoma. Low
grade. It has a good prognosis.
(Answer B)
108. Which of the following spinal tumors is located intramedullary? (April 96)
A) Neurofibroma
B) Fibrosarcoma
C) Meningioma
D) Neuroblastoma
E) Ependymoma
Ependymomas are located in the medulla spinalis at the site of the canal centralis. Sowing through CSF
metastasis. Ependymoma can be seen at any age. It may be located in the ventricle or central canal.
Intracranial ones are common in the first two decades and intraspinal ones are common in adults.
Intracranial ones are most commonly located in the 4th ventricle and cause hydrocephalus and CIBAS. They
form a well-circumscribed mass that develops from ventricular wall or central canal debris. Papillary
structure can be traced.
TUSTIME ALL TASS QUESTIONS
75
SMALL INTERNSHIP
Prolonged cells lined up in vascular circles are perivascular pseudorozet; arrayed cells around the lumen
forms ependymal badges. Although most are well differentiated, sometimes anaplastic changes may occur.
It can be used.
They may spread in the subarachnoid region. In the Kauda equina region, there is a special
myxopapillary ependymoma of the ependymoma type. Lateral and around the 4th ventricle
I dimoa; 3. colloid cyst at the base of the ventricle; choroid plexus papilloma and carcinoma
area tumors.
(Answer E)
109. Which lobe lesions are seen in Broca's aphasia? (April 96)
A) Frontal
B) Parietal
C) Occipital
D) Temporal
E) Cerebellum
The broca field innervates the larynx and mouth muscles, verbalizing the words. inferior
frontal gyrus.
(Answer A)

file:///C|/Users/User/Downloads/small%20intern.htm[06-Aug-19 12:48:59 AM]


110. Neurological disease in which muscle fasciculation is frequently observed
Which is? (April 96)
A) Myopathy
B) Myasthenia gravis
C) Amyotrophic lateral sclerosis
D) Guillan-Barre syndrome
E) Myotonic dystrophy
Motor neurons coming from the spinal cord to the muscle are the second motor neurons. 2. in motor neuron
damage
fasciculation occurs in the muscle. Therefore, muscle fasciculation is the basis of the 2nd motor neuron
diseases.
The findings.
Denervation hypersensitivity
In the second motor neuron damage, the amount of acetylcholine to the muscle decreases. Therefore
decreasing
number of nicotinic receptors present in skeletal muscle to better bind acetylcholine
increases.
This is called up-regulation. Increased receptors can bind even small amounts of acetylcholine.
In this case, tremors occur in the muscle.
These vibrations are called fasciculation. Denervation of this event in physiology
called hypersensitivity.
(Answer C)
76
TUSTIME ALL TASS QUESTIONS
NEUROLOGY - NEUROSURGERY
111. What is the main pathological disorder in multiple sclerosis? (April 96)
A) Endothelial injury
B) Axon degeneration
C) Vasculitis
D) Demyelination
E) Neuron degeneration
MULTIPLE SCLEROSIS The most common demyelinating disease of the central nervous system. Myelin basic
protein (MBP) is the most important antigen for T cells. MBP-reactive T lymphocytes are found in the blood
and CSF of the patients. In the sera of MS patients, it is also possible against myelin components (myelin
oligodendrocyte glyco-
protein-MOG) antibodies. Optic nerve lesions are among the most common initial findings in MS. I. Motor
neurons, sensory neurons, autonomic CNS neurons. 2. motor neuron, peripheral
does not hold nerves. Presence of plaques on MR, evoked potentials, demonstration of oligoclonal bands in
CSF
laboratory findings supporting the diagnosis of multiple sclerosis
Treatment: Glucocorticoid, plasma Exchange in acute attack
used. MTX, cyclophosphamide, IVIG may also be used for this purpose. INF-Beta is used to maintain
remission. can be used in cases. INF-Beta reduces T cell proliferation and suppresses inflammation with
immunomodulatory effect. Glatiramer acetate is a compound consisting of a random combination of 4 amino
acids (1-lysine, 1-glutamate, 1-alanine, 1-tyrosine). It acts by blocking the MHC molecule.
(Answer D)
112. A sixteen-year-old girl has complaints of drinking too much water, urination, and vasopressin. Physical
examination revealed bilateral heteronium hemianopsia and direct cranial x-ray showed suprasellar
calcification. What do you think about diagnosis? (September 95)
A) Hand-Schüller Christian B) Tuberous sclerosis
C) Craniopharyngioma D) Chordoma
E) Congenital Toxoplasma
Hand-Schüller-Christian disease is a syndromic form of histiocytosis x. It is characterized by exophthalmitis
as a result of infiltration of retroorbital tissue by histocytic cells, diabetes insipitus as a result of pituitary
infitration, and lytic areas in the bones of the head and vertebra as a result of bone infiltration.
(Answer C)
TUSTIME ALL TASS QUESTIONS

file:///C|/Users/User/Downloads/small%20intern.htm[06-Aug-19 12:48:59 AM]


77
SMALL INTERNSHIP
ALL TUSK QUESTIONS. Which imaging method is best used for orbital fractures? (September-95) A) Town
grafi
B) Caldwell
C) MR
D) CT
E) Water's
The best imaging study is bone x-ray and CT. Since the orbita is formed by quite a lot of bones,
superposition is more. Cross-sectional tomographic imaging is most useful in this region.
Left orbital floor fracture at CT
(Answer D)
114. 35 years old with sudden onset headache, stiff neck and 3rd cranial nerve paralysis
37 ° C fever was measured in the patient. What is the most likely diagnosis? (September 95)
A) Brain abscess
B) Meningitis
C) Subarachnoid hemorrhage
D) Glioma
E) Tetanus
(Answer C)
115. In case of increased intracranial pressure, which of the following
Reduces? (September 95)
A) Hyperventilation
B) Hypercapnia
C) Hypoxia
D) Acidosis
E) Hypertension
Carbon dioxide forms vasodilation in the brain vessels. For this reason, hypocarbia, which occurs as a result
of hyperventilation, makes vasocontraction in the brain vessels. Therefore, intracranial pressure decreases.
This method is used to reduce ICP in head trauma. (Moderate mechanical hyperventilation)
(Answer A)
NEUROLOGY - NEUROSURGERY
116. What is the most common cause of optic neuritis? (September 95)
A) Smoking
B) Multiple sclerosis
C) Alcohol
D) Encephalitis
E) Metastatic tumor
The most common cause of optic neuritis is multiple sclerosis, the most common demyelinating disease of
CNS.
(Answer B)
117. Which drug is used in the treatment of Parkinson's? (April 95)
A) Chlorpromazine
B) Reserpine
C) Haloperidol
D) Dopamine
E) Amantadine
ANTIPARKINSON DRUGS Normally there is a dopaminergic / cholinergic balance in the basal ganglia in the
CNS. If this
If the equilibrium changes to decrease in dopamine or increase in acetylcholine, the Parkinson's picture
occurs.
In this case, the treatment should be done; Or to increase dopaminergic activity.
Or to reduce cholinergic activity in the CNS.
DOPAMINERGIC EFFECTIVE DRUGS
Dopamine precursor Levodopa
bromocriptine
DA receptor agonists

file:///C|/Users/User/Downloads/small%20intern.htm[06-Aug-19 12:48:59 AM]


Lizurid
Pergolid Apomorphine
Increases DA release Amantadine
MAOB inhibitors Selegiline
COMT inhibitors Tolkapone / Entakapone
Levodopa (L-DOPA):
L-dopa analogue and precursor to dopamine (dopamine directly into the central nervous system
precursor is used).
Peripheral dopa decarboxylase enzyme also converts Ldopa to dopamine. This is an unwanted
It is the case. Because L dopa passes less into the central nervous system and its effect is significantly
reduced.
To reduce this conversion of levodopa at the periphery, dopa decarboxylase
should be used in combination with drugs.
TUSTIME ALL TASS QUESTIONS
79
SMALL INTERNSHIP
Dopa Decarboxylase Inhibitors
• Carbidopa
• Benserazide
Pyridoxine is a cofactor of decarboxylase enzymes. Dodamine cycle at the periphery of levodopa
increases the central effect of levodopa.
Side Effects of Levodopa:
The most common side effect is nausea and vomiting (because it produces a warning in CTT).
dyskinesia
Coombs test positive.
Coombs Test Positivity and Autoimmune Hemolytic Anemia
• Levodopa
• Methyldopa
Contraindications:
• Malignant melanoma (precursor of dopamine melanin)
• Psychosis
Bromocriptine:
• It is an ergot alkaloid.
• It is also used in the treatment of hyperprolactinoma.
• D2 receptor agonist.
• May cause erythromelalgia.
• May cause retroperitoneal fibrosis.
Lizurid
• Activates post synaptic D2 receptors.
pergolide
• Activates post synaptic D1 and D2 receptors. It is therefore stronger than bromocriptine.
Cabergoline
• Activates D2 receptors. It is the longest effectivedopamine receptor activator.
ropinirole
• Dot and D3receptor activator without ergot alkaloid. It has more effect on D2 receptor. Used orally.
pramipexole
• It is used orally. D2 and D3 receptor activator. It has more effect on D3 receptor.
It is thought to have neuroprotective effects.
Ouinagolid
• Ergot is a D2 receptor activator without alkaloid.
Selegiline:
• It is a selective and irreversible inhibitor of MAO-B enzyme.
• It is used in the initial period of the disease.
80
TUSTIME ALL TASS QUESTIONS
NEUROLOGY - NEUROSURGERY
Entakapon / Tolkapon

file:///C|/Users/User/Downloads/small%20intern.htm[06-Aug-19 12:48:59 AM]


• COMT (Catechol-o-methyl transferase) inhibitors.
• Entakapone is a peripheral effective catecholomethyl transferase inhibitor.
Tolcapone is also a enzyme in the central nervous system COMT (catechol-o-methyl transferase) enzyme
mini blocks
Tolcapone; markedly hepatotoxic.
Amantadine:
• Glutamate block N-methyl D-aspartate receptor
• It has both dopaminergic and anticholinergic effects.
• May form Livedo reticularis.
NOTE: Other drugs that make Livedo reticularis (cutis marmarato) are quinine and quinidine. also
lupus and PAN may be seen in AFA syndrome.
PLANT ANTICHOLINERGICS
In Parkinson's, they most clearly correct tremor.
• Biperidene (Akineton)
• Benztropine
• Trihexifenidil
• Orphenadrin
• Diphenhydramine (An antihistamine)
• Bornaprin
• Dietazine
• Etopropazine
• Chlorphenoxamine
• Prosiccidine
(Answer E)
118. Head trauma, one eye deviated to the right and pupillary mydriatic and light ref-
What do you think of a patient who is not conscious? (September 94)
A) Todd's paralysis
B) Kommosyo cerebri
C) Brain edema
D) Unkal herniation
E) Subfalxian herniation
Unkal herniation is the compression of the tip of the temporal lobe through the tentorium to the 3rd nerve.
(Answer D)
TUSTIME ALL TASS QUESTIONS
81
SMALL INTERNSHIP
119. Which is the least involved in multiple sclerosis? (April 95)
A) Cerebellar system B) Corticospinal tract
C) Optic nerve D) Peripheral nerve
E) Medulla spinalis
MS is a demyelinating disease of the central nervous system. It does not involve the peripheral nervous
system. Peripheral nerve
demyelinating disease that holds the system Gullian-Barredir.
(Answer D)
120. A patient with bilateral exophthalmos, chemosis, ophthalmoplegia, fever, and headache
what do you think? (September 94)
A) Retinoblastoma
B) Temporal lobe abscess
C) Orbital cellulite
D) Cavernous sinus thrombosis
E) Caroticocavernous fistula
Cavernous sinus thrombosis is usually secondary to infections (meningitis, facial cellulitis). V.
Since ophthalmic drainage problems will occur, symptoms such as exophthalmitis, headache and fever may
occur.
(Answer D)
121. Which is not seen in a patient with normal pressure hydrocephalus? (September 94)
A) Dementia

file:///C|/Users/User/Downloads/small%20intern.htm[06-Aug-19 12:48:59 AM]


B) Urinary incontinence
C) Difficulty walking
D) Speech disorder
E) Papil edema
NORMAL PRESSURE HYDROSEFALY (MAN JUDGE SYNDROME)
Advanced age group disease. It has a slow onset and progressive course. Primary (idiopathic) and
secondary form.
Secondary Etiology:
• Aquaduct pathologies (stenosis, bifurcation, gliosis),
• Head trauma,
• Subarachnoid hemorrhage,
• Brain Tumors (glial metastatic tumors)
• Cystic Diseases (arachnoid, epithelial, porcephalic cyst),
• Infections (meningitis, encephalitis) There are three characteristic clinical triad signs
1- Gait disorder 2- Mental Changes (Dementia)
3- Urinary incontinence
82
TUSTIME ALL TASS QUESTIONS
NEUROLOGY - NEUROSURGERY
Diagnosis Computed Tomography:
Ventriculomegaly, periventricular edema Lumbar Puncture:
If the patient's clinical complaints temporarily relieve or heal, the disease is in favor. Treatment: Shunt must
be installed.
(Answer E)
122. Occipital headache, nausea and vomiting, and worse for 3 months
The patient's symptoms progressed progressively and he had papillary edema. What do you think about
diagnosis? (April 94)
A) Brain tumor
B) Subarachnoid hemorrhage
C) Migraine
D) Subdural bleeding
E) Myasthenia gravis
Symptoms of ICPF include headache, nausea, vomiting and visual impairment due to papillary edema.
KIBAS-dependent vomiting in the morning to search. The reason for this is that night breathing slows
down,
carbon dioxide rises. Therefore, vasodilation occurs in the brain vessels. This increases the intracranial
pressure further.
(Answer A)
123. Falling of right eyelid, double vision, weakness, proximal muscle weakness and
What is done to support the diagnosis in a 40-year-old patient with increased fatigue?
(April 94)
A) Electromyography
B) Determination of muscle enzymes
C) Cranial tomography
D) Magnetic resonance
E) Positron Emission Tomography
Myasthenia gravis was described. Optimal diagnosis of muscle nerve junction diseases and neuropathies
method is EMG.
(Answer A)
124. Cold water lavage to the right ear of a coma patient with a brain stem intact
What happens to the eye when done? (April 94)
A) Deviation to the same side B) Deviation to the opposite side
C) Nystagmus to the same side D) Nystagmus to the opposite side
E) Inactivity in the eyes
TUSTIME ALL TASS QUESTIONS
83

file:///C|/Users/User/Downloads/small%20intern.htm[06-Aug-19 12:48:59 AM]

Das könnte Ihnen auch gefallen